Preview (15 of 110 pages)

NCLEX RN ACTUAL EXAM TEST BANK OF REAL QUESTIONS &
ANSWERS
NCLEX 2023/2024 graded A
Part 2
QUESTION 201
A pregnant client experiences a precipitous delivery. The nursing action during a precipitous
delivery is to:
A. Control the delivery by guiding expulsion of fetus
B. Leave the room to call the physician
C. Push against the perineum to stop delivery
D. Cross client’s legs tightly
Answer: A
Explanation:
(A) Controlling the rapid delivery will reduce the risk of fetal injury and perineal lacerations.
(B) The nurse should always remain with a client experiencing a precipitous delivery.
(C) Pushing against the perineum may cause fetal distress.
(D) Crossing of legs may cause fetal distress and does not stop the delivery process.
QUESTION 202
A client is diagnosed with organic brain disorder. The nursing care should include:
A. Organized, safe environment
B. Long, extended family visits
C. Detailed explanations of procedures
D. Challenging educational programs
Answer: A
Explanation:
(A) A priority nursing goal is attending to the client’s safety and well-being. Reorient
frequently, remove dangerous objects, and maintain consistent environment.
(B) Short, frequent visits are recommended to avoid overstimulation and fatigue.
(C) Short, concise, simple explanations are easier to understand.
(D) Mental capability and attention span deficits make learning difficult and frustrating.

QUESTION 203
A premature infant needs supplemental O2 therapy. A nursing intervention that reduces the
risk of retrolental fibroplasia is to:
A. Maintain O2 at 40%
B. Maintain O2 at 40%
C. Give moist O2 at 40%
D. Maintain on 100% O2
Answer: A
Explanation:
(A) Retrolental fibroplasia is the result of prolonged exposure to high levels of O2 in
premature infants. Complications are hemorrhage and retinal detachment.
(B, C, D) O2 concentration is too high.
QUESTION 204
A 35-year-old client is admitted to the hospital with diabetic ketoacidosis. Results of arterial
blood gases are pH 7.2, PaO2 90, PaCO2 45, and HCO3 16. The nursing assessment of arterial
blood gases indicate the presence of:
A. Respiratory alkalosis
B. Respiratory acidosis
C. Metabolic alkalosis
D. Metabolic acidosis
Answer: D
Explanation:
(A) Respiratory alkalosis is determined by elevated pH and low PaCO2.
(B) Respiratory acidosis is determined by low pH and elevated PaCO2.
(C) Metabolic alkalosis is determined by elevated pH and HCO3.
(D) Metabolic acidosis is determined by low pH and HCO3.
QUESTION 205
A client presents to the psychiatric unit crying hysterically. She is diagnosed with severe
anxiety disorder. The first nursing action is to:
A. Demand that she relax
B. Ask what is the problem
C. Stand or sit next to her

D. Give her something to do
Answer: C
Explanation:
(A) This nursing action is too controlling and authoritative. It could increase the client’s
anxiety level.
(B) In her anxiety state, the client cannot rationally identify a problem.
(C) This nursing action conveys a message of caring and security.
(D) Giving the client a task would increase her anxiety. This would be a late nursing action.
QUESTION 206
A primigravida is at term. The nurse can recognize the second stage of labor by the client’s
desire to:
A. Push during contractions
B. Hyperventilate during contractions
C. Walk between contractions
D. Relax during contractions
Answer: A
Explanation:
(A) The second stage of labor is characterized by uterine contractions, which cause the client
to bear down.
(B) Slow, deep, rhythmic breathing facilitates the laboring process. Hyperventilation is
abnormal breathing resulting from loss of pain control.
(C) The client should remain on bed rest during labor.
(D) Contractions result in discomfort.
QUESTION 207
The nurse working with a client who is out of control should follow a model of intervention
that includes which of the following?
A. Approach the client on a continuum of least restrictive care.
B. Challenge client’s behavior immediately with steps to prevent injury to self or others.
C. Leave the aggressive client to himself or herself, and take other clients away.
D. To ensure safety of other clients, place client in seclusion immediately when he or she
begins shouting.
Answer: A

Explanation:
(A) Approaching a client’s aggressive behavior on a continuum of least restrictive care is in
agreement with his or her rights (i.e., verbal methods to help maintain control, medication,
seclusion, and restraints, as necessary).
(B) Approaching a client in a challenging manner is threatening and inappropriate. A
nonchallenging and calm approach reflects staff in control and may increase client’s internal
control.
(C) It is inappropriate to leave an aggressive client who is acting out alone. The nurse should
acquire qualified help to prevent client from harm or injury to self or others.
(D) Moving a client to seclusion immediately for shouting is inappropriate. The nurse should
offer the client an opportunity to control self with limit setting. The client should understand
that the staff will assist with control if necessary (i.e., quietly accompany out of environment
to decrease stimulation and allow for verbalization) employing the least restrictive care model
of intervention.
QUESTION 208
A pregnant client continues to visit the clinic regularly during her pregnancy. During one of
her visits while lying supine on the examining table, she tells the RN that she is becoming
light-headed. The RN notices that the client has pallor in her face and is perspiring profusely.
The first intervention the RN should initiate is to:
A. Place the examining table in the Trendelenburg position
B. Assess the client to see if she is having vaginal bleeding
C. Obtain the client’s vital signs immediately
D. Help the client to a sitting position
Answer: D
Explanation:
(A) This position would cause the gravid uterus to bear the increased pressure of the vena
cava, which could lead to maternal hypotension, in turn causing the client to continue to have
pallor and to feel light-headed.
(B) This would not be the first intervention the RN should initiate. The RN should understand
the supine position and its effect on the gravid uterus and vena cava.
(C) The RN’s first intervention should be one that helps to alleviate the client’s symptoms.
Obtaining her vital signs will not alleviate her symptoms.

(D) This would move the gravid uterus off of the client’s vena cava, which would alleviate
the maternal hypotension that is the cause of her symptoms.
QUESTION 209
A mother who is breast-feeding her newborn asks the RN, “How can I express milk from my
breasts manually?” The RN tells her that the correct method for manual milk expression
includes using the thumb and the index finger to:
A. Alternately compress and release each nipple
B. Roll the nipple and gently pull the nipple forward
C. Slide the thumb and index finger forward from the outer border of the areola toward the
end of the nipple
D. Compress and release each breast at the outer border of the areola
Answer: D
Explanation:
(A) Manipulation of nipples will cause soreness and trauma.
(B) Pulling the nipples will cause discomfort and soreness.
(C) Sliding the thumb and index finger forward over the nipple will cause soreness.
(D) The best method to express milk from the breast is to position the thumb and index finger
at the outer border of the areola and compress. This is the location of the milk sinuses.
QUESTION 210
A client has been in labor for 10 hours. Her contractions have become hypoactive and slowed
in duration. The fetus is at 0 station, cervix is dilated 8 cm and effaced 90%. The physician
orders an oxytocin (Pitocin) infusion to be started at once. The RN begins the oxytocin
infusion. It is important that the RN discontinue the infusion if which one of the following
occur?
A. The client’s contractions are 2 minutes apart.
B. Duration of the contractions are 60 seconds.
C. The uterus relaxes between contractions.
D. The client complains that she is tired.
Answer: A
Explanation:
(A) It is very important that there is a resting phase or relaxation period between the
contractions. During this period, the uterus, placenta, and umbilical vessels re-establish blood

flow. No resting phase between contractions can lead to fetal bradycardia, fetal hypoxia, and
acidosis. It can also result in a tetanic contraction, which can cause uterine rupture.
(B) The goal of the oxytocin infusion is to help establish a contraction pattern lasting 45–60
seconds occurring every 2 minutes and a uterine tonus of 60–70 mm Hg.
(C) This choice is correct. The uterus has time to recover from the contraction.
(D) The client’s tiring is no indication to stop the infusion. She will be tired even without the
infusion.
QUESTION 211
A 2-month-old infant is receiving IV fluids with a volume control set. The nurse uses this
type of tubing because it:
A. Prevents administration of other drugs
B. Prevents entry of air into tubing
C. Prevents inadvertent administration of a large amount of fluids
D. Prevents phlebitis
Answer: C
Explanation:
(A) A volume control set has a chamber that permits the administration of compatible drugs.
(B) Air may enter a volume control set when tubing is not adequately purged.
(C) A volume control set allows the nurse to control the amount of fluid administered over a
set period.
(D) Contamination of volume control set may cause phlebitis.
QUESTION 212
A pregnant client experiences spontaneous rupture of membranes. The first nursing action is
to:
A. Assess the client’s respirations
B. Notify the physician
C. Auscultate fetal heart rate
D. Transfer to delivery suite
Answer: C
Explanation:
(A) Immediately following membrane rupture, the fetus is at risk for complications, not
necessarily the mother.

(B) The physician is notified after the nurse completes an assessment of the mother’s and
fetu’s conditions.
(C) Rupture of membranes facilitates fetal descent. A potential complication is cord prolapse,
which is assessed by auscultating fetal heart rate.
(D) Rupture of membranes does not necessarily indicate readiness to deliver.
QUESTION 213
A 30-year-old client has a history of several recent traumatic experiences. She presents at the
physician’s office with a complaint of blindness. Physical exam and diagnostic testing reveal
no organic cause. The nurse recognizes this as:
A. Delusion
B. Illusion
C. Hallucination
D. Conversion
Answer: D
Explanation:
(A) The client’s blindness is real. Delusion is a false belief.
(B) Illusion is the misrepresentation of a real, external sensory experience.
(C) Hallucination is a false sensory perception involving any of the senses.
(D) Conversion is the expression of intrapsychic conflict through sensory or motor
manifestations.
QUESTION 214
A client was not using his seat belt when involved in a car accident. He fractured ribs 5, 6,
and 7 on the left and developed a left pneumothorax. Assessment findings include:
A. Crackles and paradoxical chest wall movement
B. Decreased breath sounds on the left and chest pain with movement
C. Rhonchi and frothy sputum
D. Wheezing and dry cough
Answer: B
Explanation:
(A) Crackles are caused by air moving through moisture in the small airways and occur with
pulmonary edema. Paradoxical chest wall movement occurs with flail chest when a segment
of the thorax moves outward on inspiration and inward on expiration.

(B) Decreased breath sounds occur when a lung is collapsed or partially collapsed. Chest pain
with movement occurs with rib fractures.
(C) Rhonchi are caused by air moving through large fluid-filled airways. Frothy sputum may
occur with pulmonary edema.
(D) Wheezing is caused by fluid in large airways already narrowed by mucus or
bronchospasm. Dry cough could indicate a cardiac problem.
QUESTION 215
An 11-year-old boy has received a partial-thickness burn to both legs. He presents to the
emergency room approximately 15 minutes after the accident in excruciating pain with
charred clothing to both legs. What is the first nursing action?
A. Apply ice packs to both legs.
B. Begin débridement by removing all charred clothing from wound.
C. Apply Silvadene cream (silver sulfadiazine).
D. Immerse both legs in cool water.
Answer: D
Explanation:
(A) Ice creates a dramatic temperature change in the tissue, which can cause further thermal
injury.
(B) Charred clothing should not be removed from wound first. This creates further tissue
damage. Débridement is not the first nursing action.
(C) Applying silver sulfadiazine cream first insulates heat in injured tissue and increases
potential for infection.
(D) Emergency care of a thermal burn is immersing both legs in cool water. Cool water
permits gradual temperature change and prevents further thermal damage.
QUESTION 216
The physician is preparing to induce labor on a 40-week multigravida. The nurse should
anticipate the administration of:
A. Oxytocin (Pitocin)
B. Progesterone
C. Vasopressin (Pitressin)
D. Ergonovine maleate
Answer: A

Explanation:
(A) Oxytocin is a hormone secreted by the neurohypophysis during suckling and parturition
that produces strong uterine contractions.
(B) Progesterone has a quiescence effect on the uterus.
(C) Vasopressin is an antidiuretic hormone that promotes water reabsorption by the renal
tubules.
(D) Ergonovine produces dystocia as a result of sustained uterine contractions.
QUESTION 217
A pregnant client during labor is irritable and feels the urge to vomit. The nurse should
recognize this as the:
A. Fourth stage of labor
B. Third stage of labor
C. Transition stage of labor
D. Second stage of labor
Answer: C
Explanation:
(A) The fourth stage begins after expulsion of the placenta. Client symptoms are: fatigue;
chills; scant, bloody vaginal discharge; and nausea.
(B) The third stage is from birth to expulsion of placenta. Client symptoms are uterine
contractions, gush of blood, and perineal pain.
(C) The transition stage is characterized by strong uterine contractions and cervical dilation.
Client symptoms are irritability, restlessness, belching, muscle tremors, nausea, and vomiting.
(D) The second stage is characterized by full dilation of cervix. Client symptoms are perineal
bulge, pushing with contractions, great irritability, and leg cramps.
QUESTION 218
A physician’s order reads: Administer KCl 10% oral solution 1.5 mL. The KCl bottle reads 20
mEq/15 mL. What dosage should the nurse administer to the infant?
A. 1 mEq
B. 1.13 mEq
C. 2 mEq
D. Not enough information to calculate
Answer: C

Explanation:
(A) This answer is a miscalculation.
(B) This answer is a miscalculation.
(C) 1.33 mEq = 1 mL, then 1.5 mL X = 1.99, or 2 mEq.
(D) Information is adequate for calculation.
QUESTION 219
At 38 week’s gestation, a client is in active labor. She is using her Lamaze breathing
techniques. The RN is coaching her breathing and encouraging her to relax and work with her
contractions. Which one of the following complaints by the client will alert the RN that she is
beginning to hyperventilate with her breathing?
A. “I am cold.”
B. “I have a backache.”
C. “I feel dizzy.”
D. “I am nauseous.”
Answer: C
Explanation:
(A) Cold is not a symptom of hyperventilation. This could be due to the temperature of the
room.
(B) Backache is not a symptom of hyperventilation. This is probably due to the gravid uterus
and its effect on the back muscles, or it may be due to the client’s position in bed.
(C) Dizziness is the first symptom of hyperventilation. It occurs because the body is
eliminating too much CO2.
(D) Nausea is not a symptom of hyperventilation. It could be a symptom of pain.
QUESTION 220
While the RN is assessing a mother’s perineum on her 2nd postpartum day after having a
vaginal delivery, the RN notes a large ecchymotic area located to the left of the mother’s
perineum. Which one of the following interventions should the RN initiate at this time?
A. Have the client expose the area to air.
B. Apply ice to the perineum.
C. Encourage the client to take warm sitz baths.
D. Inform the physician.
Answer: C

Explanation:
(A) The area is bruised and painful. This action would do nothing to help with the healing
process of the perineum or to provide comfort.
(B) Ice is effective immediately after birth to reduce edema and discomfort, but not on the
2nd postpartum day.
(C) Sitz baths are useful if the perineum has been traumatized, because the moist heat
increases circulation to the area to promote healing, relaxes tissue, and decreases edema.
(D) The physician is not notified of bruising, but if a hematoma is present, then the physician
is notified.
QUESTION 221
A 16-year-old client with anorexia nervosa is on an inpatient psychiatric unit. She has a fear
of gaining weight and is refusing to eat sufficient amounts to maintain body weight for her
age, height, and stature. To assist with the problem of powerlessness and plan for the client to
no longer need to withhold food to feel in control, the nurse uses the following strategy:
A. Establish a structured environment with routine tasks and activities. Also, serve meals at
the same time each day.
B. Distract the client during meals to get her to eat because she must take in sufficient
amounts to keep from starving.
C. Do frequent room checks to be sure that the client is not hiding food or throwing it away.
D. Listen attentively and participate in in-depth discussions about food, because these actions
may encourage her to eat.
Answer: A
Explanation:
(A) Anorexia nervosa clients feel out of control. Providing a structured environment offers
safety and comfort and can help them to develop internal control, thus reducing their need to
control by self-starvation.
(B) Distraction does not focus on the client’s need for control.
(C) Doing frequent room checks reinforces feelings of powerlessness and the need to
continue with the dysfunctional behavior.
(D) Participating in long discussions about food does not make the client want to eat, but
rather this strategy allows her to indulge in her preoccupation and to continue with the
dysfunctional behavior.

QUESTION 222
A 4-year-old child has Down syndrome. The community health nurse has coordinated a
special preschool program. The nurse’s primary goal is to:
A. Provide respite care for the mother
B. Facilitate optimal development
C. Provide a demanding and challenging educational program
D. Prepare child to enter mainstream education
Answer: B
Explanation:
(A) Respite care for the family may be needed, but it is not the primary goal of a preschool
program.
(B) Facilitation of optimal growth and development is essential for every child.
(C) A demanding and challenging educational program may predispose the child to failure.
Children with retardation should begin with simple and challenging educational programs.
(D) Mental retardation associated with Down syndrome may not permit mainstream
education. A preschool program’s primary goal is not preparation for mainstream education
but continuation of optimal development.
QUESTION 223
A 3-month-old infant has had a unilateral cleft lip repair. He has resumed feedings of oral
formula. The nurse should feed the infant with:
A. Gavage tube
B. Nipple and bottle
C. A straw and cup
D. Syringe
Answer: D
Explanation:
(A) A gavage tube may damage suture line. It is the most invasive and should be the last
measure.
(B) A nipple and bottle require sucking, which may damage sutures.
(C) A 3- month-old infant is not able to drink from a straw.
(D) A syringe allows for the formula to be placed to the side and back of the mouth. This
minimizes the amount of sucking needed.

QUESTION 224
Painless vaginal bleeding in the last trimester may be caused by:
A. Menstruation
B. Abruptio placentae
C. Placenta previa
D. Polyhydramnios
Answer: C
Explanation:
(A) Menstruation should not occur during pregnancy.
(B) Abruptio placentae is marked by painful vaginal bleeding following a premature placental
detachment after 20th week of gestation.
(C) A low-lying placenta separates from the uterine wall as the uterus contracts and cervix
dilates. This separation causes painless bleeding in the 7th-8th month.
(D) Polyhydramnios is excessive amniotic fluid.
QUESTION 225
A 66-year-old female client has smoked 2 packs of cigarettes per day for 20 years. Her
arterial blood gases on room air are as follows: pH 7.35; PO2 70 mm Hg; PCO2 55 mm Hg;
HCO3 32 mEq/L. These blood gases reflect:
A. Compensated metabolic acidosis
B. Compensated respiratory acidosis
C. Compensated respiratory alkalosis
D. Uncompensated respiratory acidosis
Answer: B
Explanation:
(A) In compensated metabolic acidosis, the pH level is normal, the PCO2 level is decreased,
and the HCO3 level is decreased. The client’s primary alteration is an inability to remove
excess acid via the kidneys. The lungs compensate by hyperventilating and decreasing PCO2.
(B) In compensated respiratory acidosis, the pH level is normal, the PCO2 level is elevated,
and the HCO3 level is elevated. The client’s primary alteration is an inability to remove CO2
from the lungs, so over time, the kidneys increase reabsorption of HCO3 to buffer the CO2.
(C) In compensated respiratory alkalosis, the pH level is normal, the PCO2 level is decreased,
and the HCO3 level is decreased. The client’s primary alteration is hyperventilation, which
decreases PCO2. The client compensates by increasing the excretion of HCO3 from the body.

(D) In uncompensated respiratory acidosis, the pH level is decreased, the PCO2 level is
increased, and the HCO3 level is normal. The client’s primary alteration is an inability to
remove CO2 from the lungs. The kidneys have not compensated by increasing HCO3
reabsorption.
QUESTION 226
The client has been in active labor for the last 12 hours. During the last 3 hours, labor has
been augmented with oxytocin because of hypoactive uterine contractions. Her physician
assesses her cervix as 95% effaced, 8 cm dilated, and the fetus is at 0 station. Her oral
temperature is 100.2°F at this time. The physician orders that she be prepared for a cesarean
delivery. In preparing the client for the cesarean delivery, which one of the following
physician’s orders should the RN question?
A. Administer meperidine (Demerol) 100 mg IM 1 hour prior to the delivery.
B. Discontinue the oxytocin infusion.
C. Insert an indwelling Foley catheter prior to delivery.
D. Prepare abdominal area from below the nipples to below the symphysis pubis.
Answer: A
Explanation:
(A) Meperidine is a narcotic analgesic medication that crosses the placental barrier and
reaches the fetus, causing respiratory depression in the fetus. A narcotic medication should
never be included in the preoperative order for a cesarean delivery.
(B) Oxytocin infusion would be discontinued if client is being prepared for a cesarean
delivery because the medication would not be needed.
(C) The bladder is always emptied prior to and during the surgical intervention to prevent the
urinary bladder from accidentally being incised while the uterine incision is made.
(D) The abdominal area is always prepared to rid the area of hair before the abdominal
incision is made. Abdominal hair cannot be sterilized and could become a source for
postoperative incisional infection.
QUESTION 227
A 17-year-old client has a T-4 spinal cord injury. At present, he is learning to catheterize
himself. When he says, “This is too much trouble”. I would rather just have a Foley.’’ An
appropriate response for the RN teaching him would be:

A. “I know. It is a lot to learn. In the long run, though, you will be able to reduce infections if
you do an intermittent catheterization program.”
B. “It is not too much trouble. This is the best way to manage urination.”
C. “OK. I’ll ask your physician if we can replace the Foley.”
D. “You need to learn this because your doctor ordered it.”
Answer: A
Explanation:
(A) This response acknowledges the client’s feelings, gives him factual information, and
acknowledges that the final decision is his.
(B) This response is judgmental and discourages the client from expressing his feelings about
the procedure.
(C) Catheterization is a procedure that takes time to learn, but which, for the spinal cord–
injured client, can significantly reduce the incidence of urinary tract infections. A young
client with a T-4 injury has the hand function to learn this procedure fairly easily.
(D) The final decision about bladder elimination management ultimately rests with the client
and not the physician.
QUESTION 228
A cardinal symptom of the schizophrenic client is hallucinations. A nurse identifies this as a
problem in the category of:
A. Impaired communication
B. Sensory-perceptual alterations
C. Altered thought processes
D. Impaired social interaction
Answer: B
Explanation:
(A) Impaired communication refers to decreased ability or inability to use or understand
language in an interaction.
(B) In sensory-perceptual alterations an individual has distorted, impaired, or exaggerated
responses to incoming stimuli (i.e., a hallucination, which is a false sensory perception that is
not associated with real external stimuli).
(C) An altered thought processes problem statement is used when an individual experiences a
disruption in cognitive operations and activities (i.e., delusions, loose associations, ideas of
reference).

(D) In impaired social interaction, the individual participates too little or too much in social
interactions.
QUESTION 229
A client calls the prenatal clinic to schedule an appointment. She states she has missed three
menstrual periods and thinks she might be pregnant. During her first visit to the prenatal
clinic, it is confirmed that she is pregnant. The registered nurse (RN) learns that her last
menstrual period began on June 10. According to Nägele’s rule, the estimated date of
confinement is:
A. March 17
B. June 3
C. August 30
D. January 10
Answer: A
Explanation:
(A) Using Nägele’s rule, count back 3 calendar months from the first day of the last menstrual
period. The answer is March 10. Then add 7 days and 1 year, which would be March 17 of
the following year.
(B, C, D) This date is incorrect.
QUESTION 230
A 55-year-old client is unconscious, and his physician has decided to begin tube feeding him
using a smallbore silicone feeding tube (Keofeed, Duo-Tube). After the tube is inserted, the
nurse identifies the most reliable way to confirm appropriate placement is to:
A. Aspirate gastric contents
B. Auscultate air insufflated through the tube
C. Obtain a chest x-ray
D. Place the tip of the tube under water and observe for air bubbles
Answer: C
Explanation:
(A) Aspiration of gastric contents is usually a reliable way to verify tube placement. However,
if the client has dark respiratory secretions from bleeding, tube feedings could be mistaken
for respiratory secretions; in other words, aspirating an empty stomach is less reliable in this

instance. In addition, it is common for small-bore feeding tubes to collapse when suction
pressure is applied.
(B) Insufflation of air into large-bore nasogastric tubes can usually be clearly heard. In smallbore tubes, it is more difficult to hear air, and it is difficult to distinguish between air in the
stomach and air in the esophagus.
(C) A chest x-ray is the most reliable means to determine placement of small-bore nasogastric
tubes.
(D) Observing for air bubbles when the tip is held under water is an unreliable means to
determine correct tube placement for all types of nasogastric tubes. Air may come from both
the respiratory tract and the stomach, and the client who is breathing shallowly may not force
air out of the tube into the water.
QUESTION 231
A client had a vaginal delivery 3 days ago and is discharged from the hospital on the 2nd day
postpartum. She told the RN, “I need to start exercising so that I can get back into shape.”
Could you suggest an exercise I could begin with?’’ The RN could suggest which one of the
following?
A. Push-ups
B. Jumping jacks
C. Leg lifts
D. Kegel exercises
Answer: D
Explanation:
(A, B, C) This exercise is too strenuous at this time.
(D) This exercise is recommended for the first few days after delivery. It helps to stimulate
muscle tonus in the area of the perineum and the area around the urinary meatus and vagina.
QUESTION 232
A client had a myocardial infarction 5 days ago. His physician has ordered an
echocardiogram to determine how his myocardial infarction has affected his ventricular wall
motion. When the client asks if this test is painful, an appropriate response is:
A. “No, but you must be able to ride on a stationary bicycle while the test is being
performed.”
B. “No, but you will have to lie still and the gel that is used may be cool.”

C. “Yes, but your physician will be there and will order pain medicine for you.”
D. “Your physician has ordered medicine, which you will be given before you go for the test,
which will make you sleepy.”
Answer: B
Explanation:
(A) Riding a stationary bicycle or walking on a treadmill is done during a stress test.
(B) During an echocardiogram, the client must lie supine while a technician performs the test.
To perform the test, the technician uses a conductive gel and a transducer to obtain ultrasound
tracings of the heart.
(C) A physician need not be present during an echocardiogram, and it is neither invasive nor
painful.
(D) There is no premedication required for an echocardiogram.
QUESTION 233
A 13-year-old hemophiliac is hospitalized for hemarthrosis of his right knee. To relieve the
pain, the nurse should:
A. Place on bed rest; elevate and splint the right knee
B. Apply moist heat to the right knee
C. Administer aspirin for pain
D. Encourage active range of motion to right knee
Answer: A
Explanation:
(A) Immobilization, splinting, and bed rest will reduce the bleeding. Once bleeding is reduced
or stopped, the pain will subside.
(B) Moist heat causes vasodilation and bleeding. Ice or cold compresses should be applied.
(C) Aspirin decreases platelet aggregation, which causes bleeding.
(D) Active range of motion aggravates bleeding and damages the synovial sac during
bleeding episodes.
QUESTION 234
A 24-year-old client presents to the emergency department protesting “I am God”. The nurse
identifies this as a:
A. Delusion
B. Illusion

C. Hallucination
D. Conversion
Answer: A
Explanation:
(A) Delusion is a false belief.
(B) Illusion is the misrepresentation of a real, external sensory experience.
(C) Hallucination is a false sensory perception involving any of the senses.
(D) Conversion is the expression of intrapsychic conflict through sensory or motor
manifestations.
QUESTION 235
A 19-year-old client has sustained a C-7 fracture, which resulted in his spinal cord being
partially transected. By 2 week’s postinjury, his neck has been surgically stabilized, and he
has been transferred from the intensive care unit. A potential life-threatening complication the
nurse monitors the client for is:
A. Autonomic dysreflexia
B. Bradycardia
C. Central cord syndrome
D. Spinal shock
Answer: A
Explanation:
(A) Autonomic dysreflexia is the exaggerated sympathetic nervous system response to
various stimuli in the anesthetized Sympathetic stimulation results in severe, uncontrolled
hypertension, which may result in myocardial infarction or cerebral hemorrhage.
(B) Bradycardia occurs as a result of sympathetic blockade in the immediate postinjury
period. After spinal shock recedes, cardiovascular stability returns, but the client will be
bradycardiac for life.
(C) Central cord syndrome is a specific type of spinal cord injury that occurs as a result of
either hyperextension injuries or disrupted blood flow to the spinal cord.
(D) Spinal shock occurs in the immediate postinjury phase and usually resolves in
approximately 72 hours.

QUESTION 236
A physician’s order reads: 0.25 normal saline at 50 mL/hr until discontinued. The nurse is
using a micro drip tubing set. How many drops per minute should the nurse administer?
A. 1 gtt/min
B. 5 gtt/min
C. 50 gtt/min
D. 100 gtt/min
Answer: C
Explanation:
(A) This answer is a miscalculation.
(B) This answer is a miscalculation.
(C) 50 gtt/min.
(D) This answer is a miscalculation.
QUESTION 237
A client’s congestive heart failure has been treated, and he will soon be discharged. Discharge
teaching should include instruction to call the physician if he notices a 2-lb weight gain in a
24-hour period. Increased weight gain may indicate:
A. A diet too high in calories and saturated fat
B. Decreasing cardiac output
C. Decreasing renal function
D. Development of diabetes insipidus
Answer: B
Explanation:
(A) Increased calories may result in weight gain, but there is no indication in this question
that this man’s diet has changed in a way that would result in increased calories.
(B) Decreasing cardiac output stimulates the renin- angiotensin-aldosterone cycle and results
in fluid retention, which is reflected by weight gain.
(C) Decreasing renal function may result in fluid retention, but this question gives no
indication that this man has any renal problems.
(D) Profound diuresis occurs with diabetes insipidus, which results in weight loss.

QUESTION 238
The nurse is assessing and getting a history from a client treated for depression with a
monoamine oxidase (MAO) antidepressant. The most serious side effect associated with this
antidepressant and the ingestion of tyramine in aged foods may be:
A. Hypertensive crisis
B. Severe rash
C. Severe hypotension
D. Severe diarrhea
Answer: A
Explanation:
(A) The most serious adverse reactions of MAO inhibitors involve blood pressure and
ingestion of tyramine- containing foods, which may provoke a hypertensive crisis.
(B) MAO inhibitors cause adverse reactions affecting the central nervous system and serious
adverse reactions involving blood pressure.
(C) MAO inhibits false neurotransmitters (phenylalanines) and may produce hypotensive
reactions from gradual accumulation of these neurotransmitters.
(D) The most serious adverse reactions of MAO inhibitors involve blood pressure.
QUESTION 239
When planning care for the passive-aggressive client, the nurse includes the following goal:
A. Allow the client to use humor, because this may be the only way this client can express
self.
B. Allow the client to express anger by using “I messages”, such as “I was angry” when . . .,
etc.
C. Allow the client to have time away from therapeutic responsibilities.
D. Allow the client to give excuses if he forgets to give staff information.
Answer: B
Explanation:
(A) Ceasing to use humor and sarcasm is a more appropriate goal, because this client uses
these behaviors covertly to express aggression instead of being open with anger.
(B) Use of “I messages demonstrates proper use of assertive behavior to express anger instead
of passive-aggressive behavior.”

(C) Client is expected to complete share of work in therapeutic community because he has
often obstructed other’s efforts by failing to do his share.
(D) Client has used conveniently forgetting or withholding information as a passiveaggressive behavior, which is not acceptable.
QUESTION 240
A newborn is admitted to the newborn nursery with tremors, apnea periods, and poor sucking
reflex. The nurse should suspect:
A. Central nervous system damage
B. Hypoglycemia
C. Hyperglycemia
D. These are normal newborn responses to extrauterine life
Answer: B
Explanation:
(A) Central nervous system damage presents as seizures, decreased arousal, and absence of
newborn reflexes.
(B) In a diabetic mother, the infant is exposed to high serum glucose. The fetal pancreas
produces large amounts of insulin, which causes hypoglycemia after birth.
(C) Hypoglycemia is a common newborn problem. Increased insulin production causes
hypoglycemia, not hyperglycemia
(D) These are not normal adaptive behaviors to extrauterine life.
QUESTION 241
A schizophrenic client who is experiencing thoughts of having special powers states that “I
am a messenger” from another planet and can rule the earth. The nurse assesses this behavior
as:
A. Ideas of reference
B. Delusions of persecution
C. Thought broadcasting
D. Delusions of grandeur
Answer: D
Explanation:
(A) Clients experiencing ideas of reference believe that information from the environment
(e.g., the television) is referring to them.

(B) Clients experiencing delusions of persecution believe that others in the environment are
plotting against them.
(C) Clients experiencing thought broadcasting perceive that others can hear their thoughts.
(D) Clients experiencing delusions of grandeur think that they are omnipotent and have
superhuman powers.
QUESTION 242
A newborn infant is exhibiting signs of respiratory distress. Which of the following would the
nurse recognize as the earliest clinical sign of respiratory distress?
A. Cyanosis
B. Increased respirations
C. Sternal and subcostal retractions
D. Decreased respirations
Answer: C
Explanation:
(A) Cyanosis is a late clinical sign of respiratory distress.
(B) Rapid respirations are normal in a newborn.
(C) The newborn has to exert an extra effort for ventilation, which is accomplished by using
the accessory muscles of ventilation. The diaphragm and abdominal muscles are immature
and weak in the newborn.
(D) Decreased respirations are a late clinical sign. In the newborn, decreased respirations
precede respiratory failure.
QUESTION 243
After the RN is finished the initial assessment of a newborn baby and after the initial bonding
between the newborn and the mother has taken place in the delivery room, the RN will bring
the newborn to the well-baby nursery. Before the newborn is taken from the delivery room
and brought to the well-baby nursery, the RN makes sure that which of the following
interventions was completed?
A. The physician verifies the exact time of birth.
B. The nurse counts the instruments and sponges with the scrub nurse.
C. The nurse instills prophylactic ointment in the conjunctival sacs of the newborn’s eyes.
D. The nurse makes sure the mother and her newborn have been tagged with identical bands.
Answer: D

Explanation:
(A) The delivery room personnel are responsible for verifying time of birth.
(B) The scrub and circulating nurses count sponges and instruments.
(C) This intervention is done in the nursery.
(D) Tagging the mother and infant with identical bands is of utmost importance. The mother
wears one band, and the newborn wears two. Identical numbers on the three bands provide
identification for the newborn and the birth mother. Every time the newborn is brought to the
mother after delivery, those bands are checked to be sure that the numbers are identical.
QUESTION 244
A client experiencing delusions states, “I came here because there were people surrounding
my house that wanted to take me away and use my body for science.” The best response by
the nurse would be:
A. “Describe the people surrounding your house that want to take you away.”
B. “I need more information on why you think others want to use your body for science.”
C. “There were no people surrounding your house, your relatives brought you here, and no
one really wants your body for science.”
D. “I know that must be frightening for you; let the staff know when you are having thoughts
that trouble you.”
Answer: D
Explanation:
(A) Focusing on the delusional content does not reinforce reality.
(B) Pursuing details or more information on the delusion reinforces the false belief and
further distances the client from reality.
(C) Challenging the client’s delusional system may force the client to defend it, and you
cannot change the delusion through logic.
(D) Focusing on the feeling can reinforce reality and discourage the false belief. Seeking out
staff when thoughts are troublesome can help to decrease anxiety.
QUESTION 245
On admission, the client has signs and symptoms of pulmonary edema. The nurse places the
client in the most appropriate position for a client in pulmonary edema, which is:
A. High Fowler
B. Lying on the left side

C. Sitting in a chair
D. Supine with feet elevated
Answer: A
Explanation:
(A) High Fowler position decreases venous return to the heart and permits greater lung
expansion so that oxygenation is maximized.
(B) Lying on the left side may improve perfusion to the left lung but does not promote lung
expansion.
(C) Sitting in a chair will decrease venous return and promote maximal lung expansion.
However, clients with pulmonary edema can deteriorate quickly and require intubation and
mechanical ventilation. If a client is sitting in a chair when this deterioration happens, it will
be difficult to intervene quickly.
(D) The supine with feet elevated position increases venous return and will worsen
pulmonary edema.
QUESTION 246
A 40-year-old client is admitted to the hospital for tests to diagnose cancer. Since his
admission, he has become dependent and demanding to the nursing staff. The nurse identifies
this behavior as which defense mechanism?
A. Denial
B. Displacement
C. Regression
D. Projection
Answer: C
Explanation:
(A) Denial is the disowning of consciously intolerable thoughts.
(B) Displacement is the referring of a feeling or emotion from one person, object, or idea to
another.
(C) Regression is returning to an earlier stage of development.
(D) Projection is attributing one’s own thoughts, feelings, or impulses to another person.

QUESTION 247
A 60-year-old male client was hospitalized 3 days ago with the diagnosis of acute anterior
wall myocardial infarction. Today he has been complaining of increasing weakness and
shortness of breath. Crackles in both lung bases are audible on auscultation. He is developing:
A. An extension of his myocardial infarction
B. Pneumonia
C. Pulmonary edema
D. Pulmonary emboli
Answer: C
Explanation:
(A) Extensions of his myocardial infarction would be chest pain unrelieved with
nitroglycerin, cardiac enzyme elevations, and electrocardiographic changes.
(B) Persons with pneumonia may complain of weakness and shortness of breath and have
crackles in their lung bases. However, they would also have sputum production and
leukocytosis.
(C) Persons who have had myocardial infarctions (especially anterior wall) are at risk of
developing left ventricular heart failure, which is a major cause of pulmonary edema.
Pulmonary edema is manifest by shortness of breath, weakness, and crackles on auscultation
of the lung fields.
(D) Pulmonary emboli may be accompanied by shortness of breath, weakness, and crackles.
However, the pulmonary hypertension that accompanies pulmonary emboli results in signs of
increased systemic venous pressure as well.
QUESTION 248
A female client who has chronic obstructive pulmonary disease (COPD) has presented in the
emergency department with cough productive of yellow sputum and increasing shortness of
breath. On room air, her blood gases are as follows: pH 7.30 mm Hg, PCO2 60 mm Hg, PO2
55 mm Hg, HCO3 32 mEq/L. These arterial blood gases reflect:
A. Compensated respiratory acidosis
B. Normal blood gases
C. Uncompensated metabolic acidosis
D. Uncompensated respiratory acidosis
Answer: D
Explanation:

(A) In compensated respiratory acidosis, the pH level is normal, the PCO2 level is elevated,
and the HCO3 level is elevated. The client’s primary alteration is an inability to remove CO2
from the lungs, so over time, the kidneys increase reabsorption of HCO3 to buffer the CO2.
(B) Normal ranges for arterial blood gases for adults and children are as follows: pH 7.35–
7.45, PO2 80–100 mm Hg, PCO2 35–45 mm Hg, HCO3 21–28 mEq/L.
(C) In uncompensated metabolic acidosis the pH level is decreased, the PCO2 level is normal,
and the HCO3 level is decreased. The client’s primary alteration is an inability to remove
excess acid via the kidneys. The lungs are unable to clear the increased acid.
(D) In uncompensated respiratory acidosis, the pH level is decreased, the PCO2 level is
increased, and the HCO3 level is normal. In a person with long-standing COPD, the HCO3
level will rise gradually over time to compensate for the gradually increasing PCO2 , and the
person’s pH level will be normal. When a person with COPD becomes acutely ill, the kidneys
do not have time to increase the reabsorption of HCO3, so the person’s pH level will reflect
acidosis even though the HCO3 is elevated.
QUESTION 249
A newborn has been delivered with a meningomyelocele. The nursery nurse should position
the newborn:
A. Prone
B. Supine
C. Side lying
D. Semi-Fowler
Answer: A
Explanation:
(A) The prone position reduces pressure and tension on the sac. Primary nursing goals are to
prevent trauma and infection of the sac.
(B) The supine position exerts pressure on the sac.
(C) Newborns usually cannot maintain side-lying position.
(D) The semi- Fowler position exerts pressure on the sac.
QUESTION 250
Assessment of a newborn for Apgar scoring includes observation for:
A. Pupil response
B. Respiratory rate

C. Heart rate
D. Babinski’s reflex
Answer: C
Explanation:
(A) Pupil response should be assessed but is not part of Apgar scoring.
(B) Respiratory effort is an essential part of Apgar scoring, not respiratory rate.
(C) Heart rate is the most critical component of Apgar scoring.
(D) Assessment of Babinski’s reflex is not a component of Apgar scoring.
QUESTION 251
A client suspects that she is pregnant. She reports two missed menstrual periods. The first day
of her last menstrual period was August 3. Her estimated date of confinement would be:
A. November 7
B. November 10
C. May 7
D. May 10
Answer: D
Explanation:
(A) Wrong calculation
(B) Wrong calculation
(C) Wrong calculation
(D) Nägele’s rule is: Expected Date of Confinement = Last Menstrual Period - 3 months + 7
days + 1 year
QUESTION 252
A client’s physician has prescribed theophylline (Theo- Dur) to help control the
bronchospasm associated with the client’s COPD. Instructions that should be given to the
client include:
A. “Call your physician if you develop palpitations, dizziness, or restlessness.”
B. “Cigarette smoking may significantly increase the risk for theophylline toxicity.”
C. “Take this medication on an empty stomach.”
D. “Do not take your medicine if your pulse is less than 60 beats per minute.”
Answer: A
Explanation:

(A) Indications of theophylline toxicity include palpitations, dizziness, restlessness, nausea,
vomiting, shakiness, and anorexia.
(B) Cigarette smoking significantly lowers theophylline plasma levels.
(C) Theophylline should be taken with food to decrease stomach upset.
(D) These instructions are appropriate for someone taking digoxin.
QUESTION 253
Which type of insulin can be administered by a continuous IV drip?
A. Humulin N
B. NPH insulin
C. Regular insulin
D. Lente insulin
Answer: C
Explanation:
(A) Humulin N cannot be administered IV.
(B) NPH insulin cannot be administered IV.
(C) Regular insulin is the only insulin that can be administered IV.
(D) Lente insulin cannot be administered IV.
QUESTION 254
The nurse should facilitate bonding during the postpartum period. What should the nurse
expect to observe in the taking-hold phase?
A. Mother is concerned about her recovery.
B. Mother calls infant by name.
C. Mother lightly touches infant.
D. Mother is concerned about her weight gain.
Answer: B
Explanation:
(A) This observation can be made during the taking-in phase when the mother’s needs are
more important.
(B) This observation can be made during the taking-hold phase when the mother is actively
involved with herself and the infant.
(C, D) This observation can be made during the taking-in phase.

QUESTION 255
A 67-year-old client will be undergoing a coronary arteriography in the morning. Client
teaching about post procedure nursing care should include that:
A. Bed rest with bathroom privileges will be ordered
B. He will be kept NPO for 8–12 hours
C. Some oozing of blood at the arterial puncture site is normal
D. The leg used for arterial puncture should be kept straight for 8–12 hours
Answer: D
Explanation:
(A) Bed rest will be ordered for 8–12 hours post procedure. Flexing of the leg at the arterial
puncture site will occur if the client gets out of bed, and this is contraindicated after
arteriography.
(B) The client will be able to eat as soon as he is alert enough to swallow safely and that will
depend on what medications are used for sedation during the procedure.
(C) Oozing at the arterial puncture site is not normal and should be closely evaluated.
(D) The leg where the arterial puncture occurred must be kept straight for 8–12 hours to
minimize the risk of bleeding.
QUESTION 256
A type I diabetic client delivers a male newborn. The newborn is 45 minutes old. What is the
primary nursing goal in the nursery during the first hours for this newborn?
A. Bonding
B. Maintain normal blood sugar
C. Maintain normal nutrition
D. Monitor intake and output
Answer: B
Explanation:
(A) Bonding is necessary but would not be the priority with this newborn in the nursery.
(B) The infant will be at risk for hypoglycemia because of excess insulin production.
(C) Normal nutrition is a goal for all newborns.
(D) Monitoring intake and output is necessary but is not the most critical nursing goal.

QUESTION 257
An elective saline abortion has been performed on a 3- week primigravida. Following the
procedure, the nurse should be alert for which early side effect?
A. Water satiety
B. Thirst
C. Edema
D. Diabetes insipidus
Answer: B
Explanation:
(A) If the client is experiencing water satiety, there is no more desire for water.
(B) Absorption of saline into circulation rather than into amniotic sac increases serum sodium
and desire for water.
(C) Edema can be a late side effect caused by water intoxication.
(D) Diabetes insipidus occurs as a result of deficient antidiuretic hormone.
QUESTION 258
A male client has been hospitalized with congestive heart failure. Medical management of
heart failure focuses on improving myocardial contractility. This can be achieved by
administering:
A. Digoxin (Lanoxin) 0.25 mg po every day
B. Furosemide (Lasix) 40 mg po every morning
C. O2 2 L/min via nasal cannula
D. Nitroglycerin (Nitrol) 1 inch topically every 4 hours
Answer: A
Explanation:
(A) Digoxin is a cardiac glycoside given to clients in heart failure to improve their
myocardial contractility.
(B) Furosemide is a loop diuretic given to clients in heart failure to promote diuresis.
(C) O2 is given to clients in heart failure to increase oxygenation and to prevent or treat
hypoxemia.
(D) Nitroglycerin is a nitrate given to clients in heart failure to increase their cardiac output
by decreasing the peripheral resistance that the left ventricle must pump against.

QUESTION 259
A client is 2 hours post ventriculoperitoneal shunt placement. How should the nurse position
the client?
A. Head of bed elevated 30 degrees on nonoperative side
B. Head of bed elevated 30 degrees on operative side
C. Bed flat on operative side
D. Bed flat on nonoperative side
Answer: D
Explanation:
(A) Elevation of head on nonoperative side would be the position for the late postoperative
period.
(B) Positioning on operative side puts pressure on the suture lines and on the shunt valve.
Elevation of head in immediate postoperative period may cause rapid reduction of
cerebrospinal fluid.
(C) Placement on operative side puts pressure on the suture lines and shunt valve.
(D) Flat position on nonoperative side in the immediate postoperative period prevents
pressure on shunt valve and rapid reduction in cerebrospinal fluid.
QUESTION 260
A first-trimester primigravida is diagnosed with anemia.
The nurse should suspect that this anemia is a result of:
A. Mother’s increased blood volume
B. Mother’s decreased blood volume
C. Fetal blood volume increase
D. Increase in iron absorption
Answer: A
Explanation:
(A) Maternal blood volume increases at the end of the first trimester leading to a dilutional
anemia.
(B) Maternal blood volume increases.
(C) Fetal blood volume is minimal in the first trimester.
(D) Increased iron absorption would facilitate the manufacturing of erythrocytes and decrease
anemia.

QUESTION 261
A client was admitted with rib fractures and a pneumothorax, which were sustained as a result
of a motor vehicle accident. A chest tube was placed on the left side to reinflate his lung, and
he was transferred to a client unit. Twenty-four hours after admission he continues to have
bloody sputum, develops increasing hypoxemia, and his chest x-ray shows patchy infiltrates.
The nurse analyzes these symptoms as being consistent with:
A. Pneumonia
B. Pulmonary contusions
C. Pulmonary edema
D. Tension pneumothorax
Answer: B
Explanation:
(A) Pneumonia may be reflected by patchy infiltrates. In addition, fever, an increasing white
blood cell count, and copious sputum production would be present.
(B) Blunt chest trauma causes a bruising process in which interstitial and alveolar edema and
hemorrhage occur. This is manifest by gradual deterioration over 24 hours of arterial blood
gases and the continued production of bloody sputum. Patchy infiltrates are evident on chest
Xray 24 hours postinjury.
(C) Pulmonary edema usually results from left heart failure. It is manifest by pink, frothy
sputum; increasing dyspnea; tachycardia; and crackles on auscultation.
(D) Tension pneumothorax is a potential complication for someone with rib fractures and a
chest tube. It is manifest by diminished breath sounds on the affected side, rapidly
deteriorating arterial blood gases in the presence of an open airway, and shock that is
unexplained by other injuries.
QUESTION 262
At 30 week’s gestation, a client is admitted to the unit in premature labor. Her physician
orders that an IV be started with 500 mL D5W mixed with 150 mg of ritodrine stat. The RN
prepares the IV solution with the medication. The RN knows that clients receiving the
medication ritodrine IV should be observed closely for which one of the following side
effects:
A. Hypoglycemia
B. Hyperkalemia

C. Tachycardia
D. Increase in hematocrit and hemoglobin
Answer: C
Explanation:
(A) Ritodrine is a sympathomimetic 2-adrenergic agonist that can cause an elevation of blood
glucose and plasma insulin in pregnant women. Hyperglycemia can occur in women with
abnormal carbohydrate metabolism because of their inability to release more insulin.
(B) Hypokalemia can occur resulting from the action of the mimetics. It results from a
displacement of the extracellular potassium into the intracellular space.
(C) Ritodrine causes vasodilation of vessel walls, which can lead to hypotension. The body
compensates by increasing heart rate and pulse pressure.
(D) There is a lowering of serum iron resulting from the action of mimetics to activate
hematopoiesis.
QUESTION 263
Following a vaginal delivery, the postpartum nurse should observe for:
A. Dystocia, kraurosis
B. Chadwick’s sign
C. Fatigue, hemorrhoids
D. Hemorrhage and infection
Answer: D
Explanation:
(A) Dystocia is difficult labor. The delivery has occurred. Kraurosis is atrophy and dryness of
skin and any mucous membrane (vulva).
(B) Chadwick’s sign is a bluish color of vaginal mucosa suggestive of pregnancy.
(C) Fatigue is a common symptom in the postpartal period. Hemorrhoids may occur with
pregnancy.
(D) Hemorrhage and infection are potential complications of vaginal delivery. Hemorrhage
may result from retained placental fragments or soft uterus. Infection may occur from the
introduction of organisms into the uterus during the delivery.
QUESTION 264
A physician’s order reads: Administer furosemide oral solution 0.5 mL stat. The furosemide
bottle dosage is 10 mg/mL. What dosage of furosemide should the nurse give to this infant?

A. 5 mg
B. 0.5 mg
C. 0.05 mg
D. 20 mg
Answer: A
Explanation:
(A) 1 mg = 0.1 mL, then 0.5 mL X = 55 mg.
(B) This answer is a miscalculation.
(C) This answer is a miscalculation.
(D) This answer is a miscalculation.
QUESTION 265
A 16-month-old infant is being prepared for tetralogy of Fallot repair. In the nursing
assessment, which lab value should elicit further assessment and requires notification of
physician?
A. pH 7.39
B. White blood cell (WBC) count 10,000 WBCs/mm3
C. Hematocrit 60%
D. Bleeding time of 4 minutes
Answer: C
Explanation:
(A) Normal pH of arterial blood gases for an infant is 7.35–7.45.
(B) Normal white blood cell count in an infant is 6,000–17,500 WBCs/mm3.
(C) Normal hematocrit in infant is 28%–42%. A 60% hematocrit may indicate polycythemia,
a common complication of cyanotic heart disease.
(D) Normal bleeding time is 2–7 minutes.
QUESTION 266
Nursing care of the infant prior to surgical closure of a meningomyelocele would include:
A. Cover sac with dry sterile dressing
B. Cover sac with saline-soaked sterile dressing
C. Do not apply dressing; keep sac open to air
D. Aspirate any fluid from sac
Answer: B

Explanation:
(A) A dry, sterile dressing would adhere to the sac, causing tissue damage.
(B) A saline-soaked sterile dressing protects the sac from contamination by air and prevents
drying.
(C) A sac open to air causes drying and potential for contamination.
(D) This intervention is not an independent nursing action.
QUESTION 267
A client has consented to have a central venous catheter placed. The best position in which to
place the client is the Trendelenburg position. The reason is that the Trendelenburg position:
A. Allows the physician to visualize the subclavian vein
B. Reduces the possibility of air embolism
C. Reduces the possibility of hematoma formation
D. Makes the procedure more comfortable for the client
Answer: B
Explanation:
(A) The subclavian vein is not visible during central line insertion regardless of the client’s
position.
(B) The Trendelenburg position reduces the possibility of air embolism because it places
slight positive pressure on the central veins. It also distends the veins, and distention
facilitates insertion.
(C) This response is untrue; it has no effect on hematoma formation.
(D) This position is not necessarily more comfortable for the client, and many clients,
especially those who may be short of breath, may find the position uncomfortable and
difficult to maintain.
QUESTION 268
A client has a chest tube placed in his left pleural space to re-expand his collapsed lung. In a
closed-chest drainage system, the purpose of the water seal is to:
A. Prevent air from entering the pleural space
B. Prevent fluid from entering the pleural space
C. Provide a means to measure chest drainage
D. Provide an indicator of respiratory effort
Answer: A

Explanation:
(A) A chest tube extends from the pleural space to a collection device. The tube is placed
below the surface of the saline so that air cannot enter the pleural space.
(B) Fluid may enter the pleural space as a result of injury or disease. A chest tube may drain
fluid from the pleural space, but the water seal is not involved in this.
(C) Chest drainage should be measured, but the water seal is not involved in this.
(D) Fluctuations in the tube in the water- sealed bottle will give an indication of respiratory
effort, but that is not the purpose of the water seal.
QUESTION 269
A 3-year-old child is admitted with a diagnosis of possible noncommunicating hydrocephalus.
What is the first symptom that indicates increased intracranial pressure?
A. Bulging fontanelles
B. Seizure
C. Headache
D. Ataxia
Answer: C
Explanation:
(A) Bulging fontanelles are a symptom of increased intracranial pressure in infants.
(B) Seizure is a late sign of increased intracranial pressure.
(C) Headache is a very early symptom of increased intracranial pressure in the child.
(D) Ataxia is a late sign of increased intracranial pressure.
QUESTION 270
A client is in active labor and has been admitted to the labor and delivery unit. The RN has
just done a sterile vaginal exam and determines that the client is dilated 5 cm, effaced 85%,
and the fetus head is at 0 station. She asks if she could have a lumbar epidural now. The
epidural is started, and the anesthetic agent used is bupivacaine (Marcaine). After the client
has received her lumbar epidural, it is important for the RN to monitor her for which of the
following side effects:
A. Hypertension
B. Hypotension
C. Hypoglycemia
D. Hyperglycemia

Answer: B
Explanation:
(A) The medication bupivacaine will cause vasodilation in the vascular system, and this does
not result in elevation of the ma-ternal blood pressure.
(B) The medication bupivacaine will cause vasodilation in the vascular system, and this will
result in lowering the maternal blood pressure.
(C) Bupivacaine does not interfere with the functioning of the endocrine system.
(D) Bupivacaine does not interfere with the functioning of the endocrine system.
QUESTION 271
At 16 week’s gestation, a pregnant client is admitted to the maternity unit to have a
McDonald procedure (cerclage) done. She tells the RN who is admitting her to the unit that
her physician had explained what this procedure was, but that she did not understand. The RN
explains to the client that the purpose for this procedure is to:
A. Reinforce an incompetent cervix
B. Repair the amniotic sac
C. Evaluate cephalopelvic disproportion
D. Dilate the cervix
Answer: A
Explanation:
(A) The treatment most commonly uses the Shirodkar-Barter procedure (McDonald
procedure) or cerclage to enforce the weakened cervix by encircling it with a suture at the
level of the internal os.
(B) There is no known procedure that is used to repair the amniotic sac.
(C) Cephalopelvic disproportion is evaluated later in pregnancy. It is not related to this
procedure.
(D) No procedure is done to dilate the cervix at 16 week’s gestation unless the pregnancy is to
be terminated.
QUESTION 272
A 55-year-old man has recently been diagnosed with hypertension. His physician orders a
low-sodium diet for him. When he asks, “What does salt have to do with high blood
pressure?” the nurse’s initial response would be:
A. “The reason is not known why hypertension is associated with a high-salt diet.”

B. “Large amounts of salt in your diet can cause you to retain fluid, which increases your
blood pressure.”
C. “Salt affects your blood vessels and causes your blood pressure to be high.”
D. “Salt is needed to maintain blood pressure, but too much causes hypertension.”
Answer: B
Explanation:
(A) This response is untrue.
(B) Decreasing salt intake reduces fluid retention and decreases blood pressure.
(C) Salt does not have an effect on the blood vessels themselves, but on fluid retention, which
accompanies salt intake.
(D) This response is untrue.
QUESTION 273
A burn victim’s immunization history is assessed by the nurse. Which immunization is of
priority concern?
A. Oral poliovirus vaccine
B. Inactivated poliovirus vaccine
C. Tetanus toxoid
D. Hepatitis B vaccine
Answer: C
Explanation:
(A) Oral poliovirus vaccine is given to prevent polio. Polio is transmitted by direct contact
with an infected person.
(B) Inactivated poliovirus vaccine is given to adults and immunosuppressed individuals. Polio
is transmitted by direct contact with an infected person.
(C) Tetanus toxoid prevents tetanus. Tetanus is transmitted through contaminated wounds.
(D) Hepatitis B vaccine prevents hepatitis B infection. Hepatitis B is transmitted through
contact with infected blood or body fluids.
QUESTION 274
A 1000-mL dose of D5W 12 normal saline is to be infused in 8 hours. The drop factor for the
tubing is 60 gtt/min. How many drops per minute should the nurse administer?
A. 75 gtt/min
B. 100 gtt/min

C. 125 gtt/min
D. 150 gtt/min
Answer: C
Explanation:
(A) This answer is a miscalculation.
(B) This answer is a miscalculation.
(C) 125 gtt/min.
(D) This answer is a miscalculation.
QUESTION 275
A schizophrenic is admitted to the psychiatric unit. What affect would the nurse expect to
observe?
A. Anger
B. Apathy and flatness
C. Smiling
D. Hostility
Answer: B
Explanation:
(A) Anger is an emotion that is not necessarily present in schizophrenia.
(B) Lack of response to or involvement with environment and distancing are characteristic of
schizophrenia.
(C) Euphoria is more characteristic of manic-depressive disorder (bipolar disorder).
(D) Hostility is an emotion that is not necessarily present in schizophrenia.
QUESTION 276
A client was prescribed a major tranquilizer 2 months ago. One month ago she was placed on
benztropine (Cogentin). What would indicate that benztropine therapy is effective?
A. Smooth, coordinated voluntary movement
B. Tremors
C. Rigidity
D. Muscle weakness
Answer: A
Explanation:

(A) Benztropine is prescribed to decrease or alleviate extrapyramidal side effects of major
tranquilizers. Smooth, coordinated voluntary movement indicates minimal extrapyramidal
side effects.
(B) Tremors are an extrapyramidal side effect.
(C) Rigidity is an extrapyramidal side effect.
(D) Muscle weakness is an extrapyramidal side effect.
QUESTION 277
A male client has asthma and his physician has prescribed beclomethasone (Vanceril) 3 puffs
tid in addition to his other medications. After taking his beclomethasone, the client should be
instructed to:
A. Clean his inhaler with warm water and soak it in a 10% bleach solution
B. Drink a glass of water
C. Sit and rest
D. Use his bronchodilator inhaler
Answer: B
Explanation:
(A) Inhalers should be cleaned once a day. They should be taken apart, washed in warm
water, and dried according to manufacturer’s instructions. Soaking in bleach is inappropriate.
(B) A common side effect of inhaled steroid preparations is oral candidal infection. This can
be prevented by drinking a glass of water or gargling after using a steroid inhaler.
(C) There is nothing wrong with sitting and resting after using a steroid inhaler, but it is not
necessary.
(D) If a person is using a steroid inhaler as well as a bronchodilator inhaler, the
bronchodilator should always be used first. The reason for this is that the bronchodilator
opens up the person’s airways so that when the steroid inhaler is used next, there will be
better distribution of medication.
QUESTION 278
At 30 week’s gestation, a client is admitted to the unit in premature labor. Her contractions
are every 5 minutes and last 60 seconds, her cervix is closed, and the suture placed around her
cervix during her 16th week of gestation, when she had the MacDonald procedure, can still
be felt by the physician. The amniotic sac is still intact. She is very concerned about

delivering prematurely. She asks the RN, “What is the greatest risk to my baby if it is born
prematurely?” The RN’s answer should be:
A. Hyperglycemia
B. Hypoglycemia
C. Lack of development of the intestines
D. Lack of development of the lungs
Answer: D
Explanation:
(A) Any infant would be at risk for hyperglycemia because the infant’s liver is missing the
islets of Langerhans, which secrete insulin to break down glucose for cellular use.
Prematurity is not an added risk for hyperglycemia.
(B) Both premature and mature infants can be at risk for hypoglycemia if their mother had
gestational diabetes during pregnancy or entered the pregnancy with diabetes mellitus. These
infants are exposed to high levels of maternal glucose while in utero, which causes the islets
of Langerhans in the infant’s liver to produce insulin. After birth when the umbilical cord is
severed, the generous amount of maternal blood glucose is eliminated; however, there is
continued islet cell hyperactivity in the infant’s liver, which can lead to excessive insulin
levels and depleted blood glucose.
(C) Mature infants are born with an immature GI system. The nervous control of the stomach
is incomplete at birth, salivary glands are immature at birth, and the intestinal tract is sterile.
This is not the greatest risk to the premature infant.
(D) Infants born before 37 week’s gestation are at greatest risk for an insufficient amount of
surfactant in the alveoli system of the lungs. Surfactant helps to prevent lung collapse and
ensures stability of the respiratory system so that the infant can maintain his own respirations
once the umbilical cord is severed at birth.
QUESTION 279
A 79-year-old client with Alzheimer’s disease is exhibiting significant memory impairment,
cognitive impairment, extremely impaired judgment in social situations, and agitation when
placed in a new situation or around unfamiliar people. The nurse should include the following
strategy in the client’s care:
A. Maintain routines and usual structure and adhere to schedules.
B. Encourage the client to attend all structured activities on the unit, whether she wants to or
not.

C. Ask the client to go to an activity once. If she gives no response right away, change the
question around, asking the same thing.
D. Give the client two or three choices to decide what she wants to do.
Answer: A
Explanation:
(A) Alzheimer’s clients cope poorly with changes in routine because of memory deficits.
Schedule changes cause confusion and frustration, whereas adhering to schedules is helpful
and supports orientation.
(B) Insisting that the client go to all unit activities may antagonize her and increase her
agitation because of cognitive impairments. It may be better to allow the client time for
calming down or distraction rather than to insist that she attend every activity.
(C) When repeating a question, allow time first for a response; then use the same words the
second time to avoid further confusion.
(D) The nurse should avoid giving several choices at once. Cognitively impaired clients will
become more frustrated with making decisions.
QUESTION 280
A client has been admitted to the labor and delivery unit in active labor. After assessing her,
the RN notes that the client’s fetus position is left occipital posterior. Which of the following
statements best describes what this means to the labor process:
A. Decreases the overall time of the labor process
B. Prolongs the client’s first stage of labor
C. Decreases the time of the client’s first stage of labor
D. Prolongs the client’s third stage of labor
Answer: B
Explanation:
(A) Posterior position causes a larger diameter of the fetal head to enter the pelvis than an
anterior position. Pressure on the sacral nerves is increased, and it takes the fetus a longer
time to enter the pelvic inlet.
(B) This position will prolong the first stage of labor. When the larger diameter of the fetal
head enters the pelvis first, it will have a more difficult time accommodating to the pelvis;
therefore, it will take a longer time for the fetus to move through the pelvis.

(C) It will increase the time of labor because the larger diameter of the fetal head will have a
more difficult time accommodating to the pelvic inlet and thus will move through the pelvis
slower.
(D) In the third stage of labor the placenta is delivered; therefore, the infant has been
delivered.
QUESTION 281
On a mother’s 2nd postpartum day after having a vaginal delivery, the RN is preparing to
assess her perineum and anus as part of her daily assessment. The best position for the client
to be placed in for this assessment is:
A. Sims
B. Fowler’s
C. Prone
D. Any position that the RN chooses
Answer: A
Explanation:
(A) The Sims position is the best position for assessment of the perineum and anus. The top
leg is placed over the bottom leg, and the RN raises the upper buttocks to fully expose the
perineum and anus.
(B) Fowler’s position is a sitting position, and the perineum and anus would not be exposed.
(C) The prone position would have the mother on her back, and her perineum and anus would
not be exposed.
(D) The position of choice should always be the Sims.
QUESTION 282
A laboring client presents with a prolapsed cord. The nurse should immediately place the
client in what position?
A. Reverse Trendelenburg
B. Fowler’s
C. Trendelenburg
D. Sims
Answer: C
Explanation:

(A) Reverse Trendelenburg position increases pressure on the perineum. This position will
not relieve cord pressure.
(B) Fowler’s position increases perineal pressure. Cord pressure would not be relieved.
(C) Trendelenburg position will decrease perineal pressure. Cord compression will be
decreased and increase in fetal blood flow occurs.
(D) Sims position does not relieve pressure on cord or perineum.
QUESTION 283
After an infant is delivered by cesarean delivery and placed on the warmer, the RN dries and
assesses the infant. At 1 and 5 minutes after birth, the RN does the Apgar scoring of the
infant. The RN knows that because this infant was delivered by cesarean section, he is at
increased risk for having which one of the following:
A. Cold stress
B. Cyanosis
C. Respiratory distress syndrome
D. Seizures
Answer: C
Explanation:
(A) The infant is placed on the warmer and dried after birth. Cold stress occurs when the
infant is not dried and kept warm.
(B) The fact that this infant was born by cesarean delivery does not place him at a greater risk
for cyanosis than an infant delivered vaginally. Cyanosis occurs when infants cannot
oxygenate their blood after the umbilical cord is severed.
(C) Infants born by cesarean delivery are at a higher risk for developing respiratory distress
syndrome because these infants do not pass through the pelvis, where the chest is compressed
and fluid is able to escape from the lungs.
(D) Cesarean-delivered infants are not at greater risk for seizures than infants delivered
vaginally.
QUESTION 284
A client who is 7 months pregnant is diagnosed with pyelonephritis. The nurse anticipates the
physician ordering:
A. Oxytocin
B. Magnesium sulfate (MgSO4)

C. Ampicillin
D. Tetracycline
Answer: C
Explanation:
(A) Oxytocin is prescribed to stimulate uterine contractions.
(B) MgSO4 is a central nervous system depressant prescribed to prevent and control
convulsions related to preeclampsia.
(C) Ampicillin is a penicillin derivative with no known teratogenic effects. This is the safest
antibiotic during pregnancy.
(D) Tetracycline stains teeth yellow and is not as safe as ampicillin during pregnancy.
QUESTION 285
What is the appropriate nursing action for a child with increased intracranial pressure?
A. Head of bed elevated 45 degrees with child’s head maintained in a neutral position
B. Child lying flat
C. Head turned to side
D. Frequent visitation for stimulation
Answer: A
Explanation:
(A) Elevation of head of bed and neutral head position promote drainage of cerebrospinal
fluid.
(B) Flat position increases intracranial pressure and impedes cerebrospinal fluid drainage.
(C) Head turned to either side impedes cerebrospinal fluid drainage.
(D) Child should be in a calm, quiet environment with minimal stimulation.
QUESTION 286
A nurse is performing a vaginal exam on a client in active labor. An important landmark to
assess during labor and delivery are the ischial spines because:
A. Ischial spines are the narrowest diameter of the pelvis
B. Ischial spines are the widest diameter of the pelvis
C. They represent the inlet of birth canal
D. They measure pelvic floor
Answer: A
Explanation:

(A) The fetal descent, or station, is determined by the relationship of the presenting part to the
spine.
(B) Ischial spines are the narrowest measurement.
(C) Ischial spines measure the pelvic outlet.
(D) Pelvic floor measurement is not related to fetal descent.
QUESTION 287
A 16-year-old client reports a weight loss of 20% of her previous weight. She has a history of
food binges followed by self-induced vomiting (purging). The nurse should suspect a
diagnosis of:
A. Anorexia nervosa
B. Anorexia hysteria
C. Bulimia
D. Conversion reaction
Answer: C
Explanation:
(A) Anorexia nervosa is characterized by self-starvation.
(B) Anorexia hysteria is not a known disease or disorder.
(C) Bulimia is characterized by food binges and self-induced vomiting.
(D) Conversion reaction is a defense mechanism.
QUESTION 288
A 70-year-old client is almost finished receiving her second unit of packed red blood cells.
The client, who weighs 80 lb, has started complaining of being short of breath and now has
crackles in the bases of her lungs. After slowing or stopping the transfusion, the most
appropriate initial nursing action would be to:
A. Raise the client’s head and place her feet in a dependent position
B. Notify the physician
C. Place the client on 2 liters of O2 via nasal cannula
D. Administer furosemide (Lasix) 20 mg IV push
Answer: A
Explanation:

(A) Raising the client’s head and placing her feet in a dependent position is an independent
nursing action that can be taken to decrease venous return and to reduce pulmonary
congestion.
(B) Notifying the physician is an appropriate action that should be taken after the client is
positioned to maximize her respiratory status.
(C) Placing the client on O2 may be done with a physician’s order or according to an
institution’s standing orders; however, other actions should be taken first.
(D) Furosemide 20 mg IV push is an appropriate medication for the client, but it must be
ordered by her physician.
QUESTION 289
A 45-year-old male client was admitted to a chemical dependency treatment center following
legal problems related to alcohol abuse. He states, “I know that alcohol is a problem for some
people”, but I can stop whenever I want to. I’m never sick or miss work, and no one can
complain about me. During the initial assessment, the best response by the nurse would be:
A. “The fact is you are an alcoholic or you wouldn’t be here.”
B. “I understand it took strength to admit yourself to the unit, and I will do my part to help
you to stay alcohol- free.”
C. “If you can stop drinking when you want to, why don’t you stop?”
D. “It’s good that you can stop drinking when you want to.”
Answer: B
Explanation:
(A) Direct confrontation initially is nontherapeutic and may result in the client becoming
frustrated and wanting to leave.
(B) A positive, supportive attitude builds trust, and identifying positive strength raises selfesteem. Offering help allows the client to feel that he is not alone in dealing with problems.
(C) Asking the client why or to give an explanation for his behavior puts him in a position of
having to justify his behavior to the nurse.
(D) Giving approval or placing a value on feelings or a behavior may limit the client’s
freedom to behave in a way that may displease another. This response may lead to seeking
praise instead of progress.

QUESTION 290
A 1000-mL dose of lactated Ringer’s solution is to be infused in 8 hours. The drop factor for
the tubing is 10 gtt/mL. How many drops per minute should the nurse administer?
A. 125 gtt/min
B. 48 gtt/min
C. 20 gtt/min
D. 21 gtt/min
Answer: D
Explanation:
(A) This answer is a miscalculation.
(B) This answer is a miscalculation.
(C) This answer has not been rounded off to an even number.
(D) 20.8, or 21 gtt/min.
QUESTION 291
In client teaching, the nurse should emphasize that fetal damage occurs more frequently with
ingestion of drugs during:
A. First trimester
B. Second trimester
C. Third trimester
D. Every trimester
Answer: A
Explanation:
(A) Organogenesis occurs in the first trimester. Fetus is most susceptible to malformation
during this period.
(B) Organogenesis has occurred by the second trimester.
(C) Fetal development is complete by this time.
(D) The dangerous period for fetal damage is the first trimester, not the entire pregnancy.
QUESTION 292
A 6-year-old child is attending a pediatric clinic for a routine examination. What should the
nurse assess for while conducting a vision screening?
A. Hearing test
B. Gait

C. Strabismus
D. Papilledema
Answer: C
Explanation:
(A) Hearing should be assessed separately.
(B) Gait should be assessed separately. Client usually remains in one place for vision
screening. Gait is part of neurological assessment.
(C) Strabismus is crossing of eyes or outward deviation, which may cause diplopia or
amblyopia. It is easily assessed during vision screening.
(D) Papilledema is assessed by an ophthalmoscopic examination, which follows vision
screening. It is part of neurological assessment.
QUESTION 293
A 52-year-old client’s abdominal aortic aneurysm ruptured. She received rapid massive blood
transfusions for bleeding. One potential complication of blood administration for which she is
especially at risk is:
A. Air embolus
B. Circulatory overload
C. Hypocalcemia
D. Hypokalemia
Answer: C
Explanation:
(A) Air embolism is a potential complication of blood administration, but it is fairly rare and
can be prevented by using good IV technique.
(B) Circulatory overload is a potential complication of blood administration, but because this
client is actively bleeding, she is not at high risk for overload.
(C) Hypocalcemia is a potential complication of blood administration that occurs in situations
where massive transfusion has occurred over a short period of time. It occurs because the
citrate in stored blood binds with the client’s calcium. Another potential complication for
which this client is especially at risk is hypothermia, which can be prevented by using a blood
warmer to administer the blood.
(D) Hypokalemia is not a complication of blood administration.

QUESTION 294
A young boy tells the nurse, “I don’t like my Dad to kiss or hug my Mom.” I love my Mom
and want to marry her. The nurse recognizes this stage of growth and development as:
A. Electra complex
B. Oedipus complex
C. Superego
D. Ego
Answer: B
Explanation:
(A) The Electra complex is the erotic attachment of the female child to the father.
(B) The Oedipus complex is characterized by jealousy toward the parent of the same sex and
erotic attachment to the parent of the opposite sex.
(C) The superego as described by Freud is the part of personality that is associated with
internalized parental and societal control.
(D) The ego as described by Freud is the part of personality that is associated with reality
assessment.
QUESTION 295
A 19-year-old client fell off a ladder approximately 3 ft to the ground. He did not lose
consciousness but was taken to the emergency department by a friend to have a scalp
laceration sutured. The nurse instructs the client to:
A. Clean the sutured laceration twice a day with povidone- iodine (Betadine)
B. Remove his scalp sutures after 5 days
C. Return to the hospital immediately if he develops confusion, nausea, or vomiting
D. Take meperidine 50 mg po q4–6h prn for headache
Answer: C
Explanation:
(A) Povidone-iodine is very irritating to skin and should not be routinely used.
(B) Sutures should not be removed by the client.
(C) Confusion, nausea, vomiting, and behavioral changes may indicate increasing intracranial
pressure as a result of intracerebral bleeding.

(D) Use of a narcotic opiate such as meperidine is not recommended in clients with a possible
head injury because it may produce sedation, pupil changes, euphoria, and respiratory
depression, which may mask the signs of increasing intracranial pressure.
QUESTION 296
At 32 week’s gestation, a client is scheduled for a fetal activity test (nonstress test). She calls
the clinic and asks the RN, “How do I prepare for the test I am scheduled for?” The RN will
most likely inform her of the following instructions to help prepare her for the test:
A. “You need to know that an IV is always started before the test.”
B. “You will need to drink 6 to 8 glasses of water to fill your bladder.”
C. “Do not eat any food or drink any liquids before the test is started.”
D. “You will have to remain as still as you possibly can.”
Answer: D
Explanation:
(A) An IV line is not started in a nonstress test, because this test is used as an indicator of
fetal well-being. This test measures fetal activity and heart rate acceleration.
(B) The bladder does not have to be full prior to this test. It is not a sonogram test where a full
bladder enables other structures to be scanned.
(C) It has been proved that eating or drinking liquids prior to the test can assist in increasing
fetal activity.
(D) Any maternal activity will interfere with the results of the test.
QUESTION 297
The nurse instructs a client on the difference between true labor and false labor. The nurse
explains, “In true” labor:
A. Uterine contractions will weaken with walking.
B. Uterine contractions will strengthen with walking.
C. The cervix does not dilate.
D. The fetus does not descend.
Answer: B
Explanation:
(A) Uterine contractions increase with activity.
(B) Walking will increase the strength and regularity of uterine contractions in true labor.
(C) Uterine contractions that are strong and regular facilitate cervical dilation.

(D) Regular, strong uterine contractions, as in true labor, result in fetal descent.
QUESTION 298
After the fetal activity test (nonstress test) is completed, the RN is looking at the test results
on the monitor strip.
The RN observes that the fetal heart accelerated 5 beats/min with each fetal movement. The
accelerations lasted 15 seconds and occurred 3 times during the 20- minute test. The RN
knows that these test results will be interpreted as:
A. A reactive test
B. A nonreactive test
C. An unsatisfactory test
D. A negative test
Answer: A
Explanation:
(A) A nonstress test that shows at least two accelerations of the fetal heart rate of 15 bpm with
fetal activity, lasting 15 seconds over a 20-minute period.
(B) Reactive criteria are not met. The accelerations of the fetal heart rate are not at least 15
bpm and do not last 15 seconds. This could mean fetal well-being is compromised. Usually a
contraction stress test is ordered if the nonstress test results are negative.
(C) An unsatisfactory test means the data cannot be interpreted, or there was inadequate fetal
activity. If this happens, usually the test is ordered to be done at a later date.
(D) A negative test is a term used to describe the results of a contraction stress test.
QUESTION 299
A 70-year-old client has pneumonia and has just had a respiratory arrest. He has just been
intubated with an 8- mm endotracheal tube. During auscultation of his chest, breath sounds
were found to be absent on the left side. The nurse identifies the most likely cause of this as:
A. Inappropriate endotracheal tube size
B. Left-sided pneumothorax
C. Right mainstem bronchus intubation
D. Pneumonia
Answer: C
Explanation:
(A) Appropriate endotracheal tube sizes for adults range from 7.0–8.5 mm.

(B) Pneumothorax could be indicated by an absence of breath sounds on the affected side.
However, in a recently intubated client, the first priority would be to consider tube
malposition.
(C) During intubation, the right mainstem bronchus can be inadvertently entered if the
endotracheal tube is inserted too far. Left mainstem bronchus intubation almost never occurs
because of the angle of the left mainstem bronchus.
(D) Breath sounds for someone with pneumonia may be decreased over the areas of
consolidation. However, in a recently intubated client, the first priority would be to consider
tube malposition.
QUESTION 300
After performing a sterile vaginal exam on a client who has just been admitted to the unit in
active labor and placed on an electronic fetal monitor, the RN assesses that the fetal head is at
21 station. She documents this on the monitor strip. Fetal head at 21 station means that the
fetal head is located where in the pelvis?
A. One centimeter below the ischial spines
B. One centimeter above the ischial spines
C. Has not entered the pelvic inlet yet
D. Located in the pelvic outlet
Answer: B
Explanation:
(A) The ischial spines are located on both sides of the midpelvis. These spines mark the
diameter of the narrowest part of the pelvis that the fetus will encounter. They are not sharp
protrusions that will harm the fetus. Station refers to the relationship between the ischial
spines in the pelvis and the fetus. The ischial spines are designated at 0 station. If the
presenting part of the fetus is located above the ischial spines, a negative number is assigned,
noting the number of centimeters above the ischial spines. Therefore, 1 centimeter below the
ischial spines is designated as +1 station.
(B) See explanation in One centimeter above the ischial spines is designated as +1 station.
(C) The pelvic inlet is the first part of the pelvis that the fetus enters in routine delivery. The
midpelvis is the second part of the pelvis to be entered by the fetus. The ischial spines are
located on both sides of the midpelvis.
(D) The pelvic outlet is the last part of the pelvis that the fetus will enter. When the fetus
reaches this part of the pelvis, birth is near.

QUESTION 301
Cheyne-Stokes respiratory pattern can be associated with which of the following conditions?
A. Diabetic ketoacidosis
B. Fever
C. Increased intracranial pressure
D. Spinal meningitis
Answer: C
Explanation:
(A) Kussmaul’s respirations are associated with diabetic ketoacidosis, severe hemorrhage,
peritonitis, renal failure, and uremia.
(B) Tachypnea (respiratory rate 25 breaths/min) is often associated with fever.
(C) Cheyne-Stokes respiratory pattern is most often associated with increased intracranial
pressure secondary to changes in pressure in the cerebral and cerebellar areas.
(D) Biot’s breathing is most frequently associated with spinal meningitis.
QUESTION 302
A 47-year-old client has been admitted to the general surgery unit for bowel obstruction. The
doctor has ordered that an NG tube be inserted to aid in bowel de-compression. When
preparing to insert a NG tube, the nurse measures from the:
A. Lower lip to the shoulder to the upper sternum
B. Tip of the nose to the lower lip to the umbilicus
C. End of the tube to the first measurement line on the tube
D. Tip of the nose to the ear lobe to the xiphoid process or midepigastric area
Answer: D
Explanation:
(A) This measurement is 50 cm (48–49 cm). Fifty centimeters is considered the length
necessary for the distal end of the tube to be in place in the stomach. This measurement is too
short.
(B) This measurement is 50 cm (47–48 cm). Fifty centimeters is considered the length
necessary for the distal end of the tube to be in place in the stomach. This measurement is too
short.

(C) This measurement gives an approximate indication of the length necessary for the distal
end of the tube to be in place in the stomach, but it is not as accurate as actually measuring
the client (nose-earxiphoid).
(D) This is the correct measurement of 50 cm from the tip of the client’s nose to the tip of the
earlobe to the xiphoid process (called the NEX [nose-ear-xiphoid] measurement). It is
approximately equal to the distance necessary for the distal end of the tube to be located in
the correct position in the stomach.
QUESTION 303
Because a client is taking an MAO inhibitor, it is necessary to discuss the need for adherence
to a low-tyramine diet. Which of the following are foods that she should avoid?
A. Pickled, aged, smoked, and fermented foods
B. Fresh vegetables
C. Broiled fresh fish and fowl
D. Fresh fruit such as apples and oranges
Answer: A
Explanation:
(A) These foods may produce elevation in blood pressure when consumed during MAO
inhibition therapy.
(B) These foods have not been pickled, fermented, smoked, or aged. They contain very little,
if any, tyramine or tryptophan.
(C) As long as the meat has not been aged or smoked, it is within the dietary regimen.
(D) Fresh fruits can be consumed as desired. However, the consumption of bananas is limited.
QUESTION 304
The nurse documents a client’s surgical incision as having red granulated tissue. This
indicates that the wound is:
A. Infected
B. Not healing
C. Necrotic
D. Healing
Answer: D
Explanation:
(A) The wound is not infected. An infected wound would contain pus, debris, and exudate.

(B) The wound is healing properly.
(C) A necrotic wound would appear black or brown.
(D) The wound is healing properly and is filled with red granulated tissue and fragile
capillaries.
QUESTION 305
A 16-year-old client with a diagnosis of oppositional defiant disorder is threatening violence
toward another child. In managing a potentially violent client, the nurse:
A. Must use the least restrictive measure possible to control the behavior
B. Should put the client in seclusion until he promises to behave appropriately
C. Should apply full restraints until the behavior is under control
D. Should allow other clients to observe the acting out so that they can learn from the
experience
Answer: A
Explanation:
(A) This answer is correct. Least restrictive measures should always be attempted before a
client is placed in seclusion or restraints. The nurse should first try a calm verbal approach,
suggest a quiet room, or request that the client take “time-out before placing the client in
seclusion”, giving medication as necessary, or restraining.
(B) This answer is incorrect. A calm verbal approach or requesting that a client go to his room
should be attempted before restraining.
(C) This answer is incorrect. Restraints should be applied only after all other measures fail to
control the behavior.
(D) This answer is incorrect. Other clients should be removed from the is often very anxiety
producing for other clients to see a peer out of control. It could also lead to mass acting- out
behaviors.
QUESTION 306
A client returns to the cardiovascular intensive care unit following his coronary artery bypass
graft. In planning his care, the most important electrolyte the nurse needs to monitor will be:
A. Chloride
B. HCO3
C. Potassium
D. Sodium

Answer: C
Explanation:
(A) Chloride, HCO3, and sodium will need to be monitored, but monitoring these electrolytes
is not as important as potassium monitoring.
(B) Chloride, HCO3, and sodium will need to be monitored, but monitoring these electrolytes
is not as important as potassium monitoring.
(C) Potassium will need to be closely monitored because of its effects on the heart.
Hypokalemia could result in supraventricular tachyarrhythmias.
(D) Chloride, HCO3, and sodium will need to be monitored, but monitoring these electrolytes
is not as important as potassium monitoring.
QUESTION 307
A client is admitted to the hospital with diabetic ketoacidosis.
The emergency room nurse should anticipate the administration of:
A. Humulin N
B. Humulin R
C. Humulin U
D. Humulin L
Answer: B
Explanation:
(A) Intermediate-acting insulin is not indicated in an emergency.
(B) Regular insulin is rapid acting and indicated in an emergency situation.
(C) Long-acting insulin is not indicated in an emergency situation.
(D) Intermediate-acting insulin is not indicated in an emergency situation.
QUESTION 308
A client has received digoxin 0.25 mg po daily for 2 weeks. Which of the following digoxin
levels indicates toxicity?
A. 0.5 ng/mL
B. 1.0 ng/mL
C. 2.0 ng/mL
D. 3.0 ng/mL
Answer: D
Explanation:

(A) 0.5 ng/mL of digoxin is a subtherapeutic level, not a toxic one.
(B) 1.0 ng/mL is a therapeutic level.
(C) 2.0 ng/mL is a therapeutic level.
(D) Digoxin’s therapeutic level is 0.8–2.0 ng/mL. Digoxin’s toxic level is 2.0 ng/mL.
QUESTION 309
A male client received a heart-lung transplant 1 month ago at a local transplant center. While
visiting the nursing center to have his blood pressure taken, he complains of recent weakness
and fatigue. He also tells the nurse that he is considering stopping his cyclosporine because it
is expensive and is causing his face to become round. He fears he will catch viruses and be
more susceptible to infections. The nurse responds to this last statement by explaining that
cyclosporine:
A. Is given to prevent rejection and makes him less susceptible to infection than other oral
corticosteroids
B. Is available at discount pharmacies for a reduced price
C. Is usually not necessary after the first year following transplantation
D. May initially cause weakness, dizziness, and fatigue, but these side effects will gradually
resolve themselves
Answer: A
Explanation:
(A) Cyclosporine is the immunosuppressive drug of choice. It provides immunosuppression
but does not lower the white blood cell count; therefore, the client is less susceptible to
infection.
(B) Cyclosporine is available at discount pharmacies. The cost may be absorbed by health
insurance, or Medicare, if the client is eligible. However, this statement does not address the
entire problem verbalized by the client.
(C) Immunosuppressive agents will be taken for the client’s entire life because rejection can
occur at any time.
(D) These side effects do not necessarily resolve in time; however, the client may adapt.
QUESTION 310
Decreased pulmonary blood flow, right-to-left shunting, and deoxygenated blood reaching the
systemic circulation are characteristic of:
A. Tetralogy of Fallot

B. Ventricular septal defect
C. Patent ductus arteriosus
D. Transposition of the great arteries
Answer: A
Explanation:
(A) Tetralogy of Fallot is the most common cyanotic heart defect, which includes a VSD,
pulmonary stenosis, an overriding aorta, and ventricular hypertrophy. The blood flow is
obstructed because the pulmonary stenosis decreases the pulmonary blood flow and shunts
blood through the VSD, creating a right-to-left shunt that allows deoxygenated blood the
reach the systemic circulation.
(B) A VSD alone creates a left-to-right shunt. The pressure in the left ventricle is greater than
that of the right; therefore, the blood will shunt from the left ventricle to the right ventricle,
increasing the blood flow to the lungs. No deoxygenated blood will reach the systemic
circulation.
(C) In patent ductus arteriosus, the pressure in the aorta is greater than in the pulmonary
artery, creating a left-to-right shunt. Oxygenated blood from the aorta flows into the
unoxygenated blood of the pulmonary artery.
(D) Transposition of the great arteries results in two separate and parallel circulatory systems.
The only mixing or shunting of blood is based on the presence of associated lesions.
QUESTION 311
As a postoperative cholecystectomy client completes tomorrow’s dinner menu, the nurse
knows that one of the following meal choices will best provide the essential vitamin(s)
necessary for proper tissue healing?
A. Liver, white rice, spinach, tossed salad, custard pudding
B. Fish fillet, carrots, mashed potatoes, butterscotch pudding
C. Roast chicken, gelatin with sliced fruit
D. Chicken breast fillet in tomato sauce, potatoes, mustard greens, orange and strawberry
slices
Answer: D
Explanation:
(A) This meal choice provides more of the vitamins A, D, and K than of vitamin C.
(B) This meal choice provides more of the vitamins A, B12, and D than of vitamin C.

(C) This meal choice provides more of the vitamins A, B1 (thiamine), niacin, and
microminerals than of vitamin C.
(D) This meal choice provides foods rich in vitamin C, which are essential in tissue healing.
QUESTION 312
While changing the dressing on a client’s central line, the nurse notices redness and warmth at
the needle insertion site. Which of the following actions would be appropriate to implement
based on this finding?
A. Discontinue the central line.
B. Begin a peripheral IV.
C. Document in the nurse’s notes and notify the physician after redressing the site.
D. Clean the site well and redress.
Answer: C
Explanation:
(A) The nurse may never discontinue a central line without a physician’s order.
(B) The nurse may never initiate a peripheral IV without a physician’s order except in an
emergency situation.
(C) The nurse should always document findings and alert the physician to the findings as
well. The physician may then initiate a new central line and order the current central line to
be discontinued.
(D) Besides cleaning and redressing, the nurse should always document the findings.
QUESTION 313
The nurse has been assigned a client who delivered a 6- lb, 12-oz baby boy vaginally 40
minutes ago. The initial assessment of greatest importance for this client would be:
A. Length of her labor
B. Type of episiotomy
C. Amount of IV fluid to be infused
D. Character of the fundus
Answer: D
Explanation:
The length of labor has little bearing on the fourth stage of labor. The type of labor and
delivery is significant.

(B) The type of episiotomy will affect the client’s comfort level. However, the nurse’s
assessment and implementations center on prevention of hemorrhage during the fourth stage
of labor. The amount of bleeding from the episiotomy or hematoma formation is of higher
priority than the type of episiotomy.
(C) The amount of IV fluid to be infused is a nursing function to be attended to; however, it is
lower in priority than determining if haemorrhaging is occurring.
(D) Character of the fundus would be the priority nursing assessment because changes in
uterine tone may identify possible postpartum hemorrhage.
QUESTION 314
When assessing residual volume in tube feeding, the feeding should be delayed if the amount
of gastric contents (residual) exceeds:
A. 20 mL
B. 25 mL
C. 30 mL
D. 50 mL
Answer: D
Explanation:
(A) A residual volume of 20 mL is not excessive.
(B) A residual volume of 25 mL is not excessive.
(C) A residual volume of 30 mL is not excessive.
(D) Tube feedings should be withheld and physician notified for residual volumes of 50–100
mL.
QUESTION 315
The nurse begins morning assessment on a male client and notices that she is unable to
palpate either of his dorsalis pedis pulses in his feet. What is the first nursing action after
assessing this finding?
A. Palpate these pulses again in 15 minutes.
B. Use a Doppler to determine presence and strength of these pulses.
C. Document the finding that the pulses are not palpable.
D. Call the physician and notify the physician of this finding.
Answer: B
Explanation:

(A) Palpating these pulses again in 15 minutes may only result in the same findings.
(B) Any time during an assessment that the nurse is unable to palpate pulses, the nurse should
then obtain a Doppler and assess for presence or absence of the pulse and pulse strength, if a
pulse is present.
(C) Pulses may be present and assessed through use of a Doppler. Absence of palpable pulses
does not indicate absence of blood flow unless pulses cannot be located with a Doppler.
(D) The nurse would only call the physician after determining that the pulses are absent by
both palpation and Doppler.
QUESTION 316
A child receiving chemotherapeutic drugs experiences a loss of appetite directly related to the
therapy. Which of the following strategies should be most effective in encouraging the child
to eat?
A. Provide a well-balanced diet at usual times, and restrict dessert if the child fails to eat well.
B. Schedule procedures immediately after eating so that the child will not be tired or in pain
at mealtime.
C. Offer the child a diet with a wider variety of foods and with more seasoning than her usual
diet.
D. Offer the child smaller meals more frequently than usual, and include as many of her
favorite foods as possible.
Answer: D
Explanation:
(A) Because the child’s appetite is capricious at best, regular servings may be overwhelming.
Praise the child for what is eaten.
(B) The child will soon learn that procedures follow meals and may play with food rather
than eat it to avoid or delay the procedure.
(C) Young children usually do not like highly seasoned foods and may need the security of
usual foods. Such a change may actually increase anorexia.
(D) Small servings appear more achievable to the child, and the inclusion of favorite foods
can add a sense of security.
QUESTION 317
In admitting a client to the psychiatric unit, the nurse must explain the rules and regulations
of the unit. A client with antisocial personality disorder makes the following remark, “Forget

all those rules.” I always get along well with the nurses. Which nursing response to him
would be most effective?
A. “OK, don’t listen to the rules. See where you end up.”
B. “I’m pleased that you get along so well with the staff. You must still know and abide by
the rules.”
C. “It is irrelevant whether you get along with the nurses.”
D. “I’m not the other nurses. You better read the rules yourself.”
Answer: B
Explanation:
(A) This answer is incorrect. A nurse should be an appropriate role model. Threats are not
appropriate. No limit setting was stated.
(B) This answer is correct. The nurse made a positive statement followed by a simple, clear,
concise setting of limits.
(C) This answer is incorrect. It appears to have a negative connotation. There was no limit
setting.
(D) This answer is incorrect. The nurse obviously responded in a negative manner. Learning
takes place more readily when one is accepted, not rejected. No limits were set.
QUESTION 318
A client is having a pneumonectomy done today, and the nurse is planning her postoperative
care. Nursing interventions for a postoperative left pneumonectomy would include:
A. Monitoring the chest tubes
B. Positioning the client on the right side
C. Positioning the client in semi-Fowler position with a pillow under the shoulder and back
D. Monitoring the right lung for an increase in rales
Answer: D
Explanation:
(A) Chest tubes are usually not necessary in a pneumonectomy because there is no lung to reexpand on the operative side.
(B) The pneumonectomy client should be positioned on the back or operated side because the
sutured bronchial stump may open, allowing fluid to drain into the unoperated side and drown
the client.
(C) The client should not have a pillow under the shoulder and back because of the
subscapular incision.

(D) Rales are commonly heard over the base of the remaining lung, but an increase could
indicate circulatory overload and therefore should be closely monitored.
QUESTION 319
The nurse recognizes that a client with the diagnosis of cholecystitis and cholelithiasis would
expect to have stools that are:
A. Clay or gray colored
B. Watery and loose
C. Bright-red streaked
D. Black
Answer: A
Explanation:
(A) Clients who have obstruction in the biliary tract so that bile is not released into the
duodenum experience a change in stools from brown to gray or clay colored.
(B) This type of stool can occur with other GI problems, such as bacterial or viral infections,
and other disease problems, and is not a common finding with biliary obstructions such as
cholecystitis and cholelithiasis.
(C) This type of stool is usually associated with a GI or bowel problem, such as lower GI
bleeding, rather than with biliary obstructions.
(D) This type of stool is usually associated with a GI or bowel problem, such as upper GI
bleeding, rather than with biliary obstructions.
QUESTION 320
Following a fracture of the left femur, a client develops symptoms of osteomyelitis. During
the acute phase of osteomyelitis, nursing care is directed toward:
A. Moving or turning the client’s left leg carefully to minimize pain and discomfort
B. Allowing the client out of bed only in a wheelchair or gurney to minimize weight bearing
on the left leg
C. Providing the client with a high-protein, high-fiber diet to promote healing
D. Instituting physical therapy to ensure restoration of optimal functioning of the leg
Answer: A
Explanation:
(A) Any movement of his affected limb will cause discomfort to the child.

(B) No weight bearing will be allowed until healing is well underway to avoid pathological
fractures.
(C) The child will be anorexic and may experience vomiting. Diet should be simple and high
caloric until appetite returns and symptoms subside.
(D) Physical therapy is instituted only after infection subsides.
QUESTION 321
In working with a manipulative client, which of the following nursing interventions would be
most appropriate?
A. Bargaining with the client as a strategy to control the behavior
B. Redirecting the client
C. Providing a consistent set of guidelines and rules
D. Assigning the client to different staff persons each day
Answer: C
Explanation:
(A) This answer is incorrect. Bargaining is a manipulative act, which the nurse could expect
from the client.
(B) This answer is incorrect. Confrontation is an effective nursing strategy with manipulative
behavior. Redirection is appropriate for the client who is out of touch with reality.
(C) This answer is correct. Manipulative clients must abide by consistent rules.
(D) This answer is incorrect. Manipulation is kept at a minimum if the same staff person is
assigned to the client. Often the client will attempt to play staff persons against each other.
QUESTION 322
A 19-year-old male client arrived via ambulance to the emergency room following a
motorcycle accident. He is comatose. His face has evidence of dried blood. On assessment,
the nurse notes an obvious injury to his left eye. The preferred positioning for a client with an
obvious eye injury is:
A. Reclining to control bleeding
B. Any position in which the client is comfortable
C. Side-lying, either left or right
D. Sitting with head support
Answer: D
Explanation:

(A) A reclining position can cause a penetrating object to advance further into the eye.
(B) Prevention of further injury is the priority, not comfort.
(C) A side-lying position may increase intraocular and intracranial pressure if an
accompanying head injury is suspected.
(D) A sitting position with the head supported will prevent further injury while allowing
injury care to take place.
QUESTION 323
A 50-year-old depressed client has recently lost his job. He has been reluctant to leave his
hospital room. Nursing care would include:
A. Forcing the client to attend all unit activities
B. Encouraging the client to discuss why he is so sad
C. Monitoring elimination patterns
D. Providing sensory stimulation
Answer: C
Explanation:
(A) The client should be encouraged to attend the unit activities. The nurse and client should
choose a few activities for the client to attend that will be positive experiences for him.
(B) The nurse should encourage the client to discuss his feelings and to begin to deal with the
depression.
(C) Depressed persons often have little appetite and poor fluid intake. Constipation is
common.
(D) A calm, consistent level of stimuli is most effective. Sensory deprivation and
overstimulation should be avoided.
QUESTION 324
A 15-year-old client was diagnosed as having cystic fibrosis at 8 months of age. He is in the
hospital for a course of IV antibiotic therapy and vigorous chest physiotherapy. He has a poor
appetite. The nurse can best help him to meet the desired outcome of consuming a prescribed
number of calories by:
A. Including the client in planning sessions to select the type of meal plan and foods for his
diet
B. Working with the nutritionist to devise a diet with significantly increased calories

C. Selecting foods for the client’s diet that are high in calories and instituting a strict calorie
count
D. Constantly providing him with chips, dips, and candies, because the number of calories
consumed is more important than the quality of foods
Answer: A
Explanation:
(A) The adolescent knows what he likes and will be more likely to eat if he has some control
over his diet.
(B) The nurses and nutritionist can plan an excellent diet, but it will not help the adolescent
unless he eats it.
(C) Eating is already a chore for this client. Adding a strict calorie count could make it even
more burdensome.
(D) Fats are particularly difficult for the cystic fibrosis client to digest. He does need a
healthful diet, not just more calories.
QUESTION 325
A 68-year-old man was recently diagnosed with end stage renal disease. He has not yet begun
dialysis but is experiencing severe anemia with associated symptoms of dyspnea on exertion
and chest pain. Which statement best describes the management of anemia in renal failure?
A. Hematocrit levels usually remain slightly below normal in clients with renal failure.
B. Transfusion is often begun as early as possible to prevent complications of anemia such as
dyspnea and angina.
C. Anemia in renal failure is frequently caused by low serum iron and ferritin and corrected
by oral iron and ferritin replacement therapy.
D. The renal secretion of erythropoiesis is decreased. The bone marrow requires
erythropoietin to mature red blood cells.
Answer: D
Explanation:
(A) Clients in renal failure typically have very low hematocrits, often in the range of 16–22%.
(B) Transfusion is avoided unless the client exhibits acute symptoms such as dyspnea, chest
pain, tachycardia, and extreme fatigue. When the client is given a transfusion, the bone
marrow adjusts by producing less red blood cells.
(C) Anemia in renal failure is caused primarily by decreased erythropoietin. Low serum iron
and ferritin may aggravate the anemia and require treatment.

(D) Decreased secretion of erythropoietin by the kidney is the primary cause of anemia. The
bone marrow requires this hormone to mature red blood cells. Treatment is with replacement
therapy.
QUESTION 326
One week ago, a 21-year-old client with a diagnosis of bipolar disorder was started on lithium
300 mg po qid. A lithium level is ordered. The client’s level is 1.3 mEq/L. The nurse
recognizes that this level is considered to be:
A. Within therapeutic range
B. Below therapeutic range
C. Above therapeutic range
D. At a level of toxic poisoning
Answer: A
Explanation:
(A) This answer is correct. The therapeutic range is 1.0–1.5 mEq/L in the acute phase.
Maintenance control levels are 0.6–1.2 mEq/L.
(B, C) This answer is incorrect. A level of 1.3 mEq/L is within therapeutic range.
(D) This answer is incorrect. Toxic poisoning is usually at the 2.0 level or higher.
QUESTION 327
Chorioamnionitis is a maternal infection that is usually associated with:
A. Prolonged rupture of membranes
B. Post term deliveries
C. Maternal pyelonephritis
D. Maternal dehydration
Answer: A
Explanation:
(A) Chorioamnionitis is an inflammation of the chorion and amnion that is generally
associated with premature or prolonged rupture of membranes.
(B) Post term deliveries have not been shown to increase the risk of chorioamnionitis unless
there has been prolonged rupture of membranes.
(C) Pyelonephritis is a kidney infection that develops in 20%–40% of untreated maternal
UTIs.
(D) Maternal dehydration, though of great concern, is not related to chorioamnionitis.

QUESTION 328
A male client had a right below-the-knee amputation 4 days ago. His incision is healing well.
He has gotten out of bed several times and sat at the side of the bed. Each time after returning
to bed, he has experienced pain as if it were located in his right foot. Which nursing measure
indicates the nurse has a thorough understanding of phantom pain and its management?
A. Phantom pain is entirely in the client’s mind. The client should be instructed that the pain
is psychological and should not be treated.
B. The basis for phantom pain may occur because the nerves still carry pain sensation to the
brain even though the limb has been amputated. The pain is real, intense, and should be
treated.
C. The cause of phantom pain is unknown. The nurse should provide the client with support,
promote sleep, and handle the injured limb smoothly and gently.
D. Phantom pain is caused by trauma, spasms, and edema at the incisional site. It will
decrease when postoperative edema decreases. It should be treated with nonnarcotic
medication whenever possible.
Answer: B
Explanation:
(A) This statement is entirely false.
(B) Phantom pain may be caused by nerves continuing to carry sensation to the brain even
though the limb is removed. It is real, intense, and should be treated as ordinary pain would.
(C) Although the cause of phantom pain is still unknown, these measures may promote the
relief of any type of pain, not just phantom pain.
(D) Phantom pain is not caused by trauma, spasms, and edema and will not be relieved by
decreasing edema.
QUESTION 329
A 9-month-old infant visits her paediatrician for a routine visit. A developmental assessment
was initiated by the nurse. Which skill would cause the nurse to be concerned about the
infant’s developmental progression?
A. She sits briefly alone with assistance.
B. She creeps and crawls.
C. She pulls herself to her feet with help.
D. She stands while holding onto furniture.

Answer: A
Explanation:
(A) The 9-month-old infant can sit alone for long periods. By the age of 6 months, many
infants can pull themselves to a sitting position.
(B, C, D) This skill represents normal development.
QUESTION 330
A 70-year-old homeless woman is admitted with pneumonia. She is weak, emaciated, and
febrile. The physician orders enteral feedings intermittently by nasogastric tube. When
inserting the nasogastric tube, once the tube passes through the oropharynx, the nurse will
instruct the client to:
A. Tilt her head backwards
B. Swallow as tube passes
C. Hold breath as tube passes
D. Cough as tube passes
Answer: B
Explanation:
(A) Head should be tilted slightly forward to facilitate insertion.
(B) Swallowing assists with insertion of tube and closes off airway.
(C) Client should be swallowing as tube passes; holding the breath facilitates nothing.
(D) Coughing may expel tube.
QUESTION 331
A 29-year-old client is diagnosed with borderline personality disorder. He has aroused the
nurse’s anger by using a condescending tone of voice with other clients and staff persons.
Which of the following statements from the nurse would be most appropriate in
acknowledging feelings regarding the client’s behavior?
A. “I feel angry when I hear that tone of voice.”
B. “You make me angry when you talk to me that way.”
C. “Are you trying to get me angry?”
D. “Why do you treat me that way?”
Answer: A
Explanation:

The nurse appropriately states how he or she feels when the client speaks in a condescending
manner.
(B) This statement indicates that the client has control over the nurse. No one makes another
person angry; each individual has a choice.
(C) “Why questions usually put a person on the defensive.” In addition, the client cannot
“make the nurse angry.” The client does not have that control.
(D) Again, a “why statement places the client on the defensive.”
QUESTION 332
The nurse is assisting a 4th-day postoperative cholecystectomy client in planning her meals
for tomorrow’s menu. Which vitamin is the most essential in promoting tissue healing?
A. Vitamin C
B. Vitamin B1
C. Vitamin D
D. Vitamin A
Answer: A
Explanation:
(A) Vitamin C (ascorbic acid) is essential in promoting wound healing and collagen
formation.
(B) Vitamin B1 (thiamine) maintains normal gastrointestinal (GI) functioning, oxidizes
carbohydrates, and is essential for normal functioning of nervous tissue.
(C) Vitamin D regulates absorption of calcium and phosphorus from the GI tract and helps
prevent rickets.
(D) Vitamin A is necessary for the formation and maintenance of skin and mucous
membranes. It is also essential for normal growth and development of bones and teeth.
QUESTION 333
On admission to the inpatient unit, a 34-year-old client is able to follow simple directions, but
with great difficulty. He is worried about how he can keep clean in such a public place and
repeatedly dusts his bureau, straightens his bed, and adjusts the clothes in his closet. The
client is experiencing a severe level of anxiety. Which response by the nurse would be most
therapeutic in initially attempting to reduce his anxiety?
A. “You will not be allowed to remain in your room if you continue to bother things.”

B. “I can see how uncomfortable you are, but I would like you to walk with me so I can show
you around the unit.”
C. “Tell me why your room needs to be so clean.”
D. “I’ve inspected this room and it is perfectly clean.”
Answer: B
Explanation:
(A) This statement is punitive.
(B) Acknowledging the anxiety and channelling it into some positive activity is therapeutic.
(C) The client cannot say “why; this statement puts the client on the defensive.”
(D) A rational approach, especially a judgmental one, is nontherapeutic.
QUESTION 334
A baby who was diagnosed with pyloric stenosis has continued to have projectile vomiting.
With prolonged vomiting, the infant is prone to:
A. Respiratory acidosis
B. Respiratory alkalosis
C. Metabolic acidosis
D. Metabolic alkalosis
Answer: D
Explanation:
(A) Respiratory acidosis is the result of problematic ventilation. Plasma pH decreases, while
plasma PCO2 and plasma HCO3 increase.
(B) Respiratory alkalosis results from increased respiratory rate and depth. Plasma pH
increases, while plasma PCO2 and plasma HCO3 decrease.
(C) Metabolic acidosis occurs when there is strong acid gain in the body. Plasma pH, PCO2,
and HCO3 decrease.
(D) Increased risk for metabolic alkalosis is due to a loss of hydrogen ions; depletion of
potassium, sodium, and chloride when vomiting occurs. Plasma pH and plasma PCO2
increase; plasma HCO3 may decrease and then increase to compensate.
QUESTION 335
A client with cystic fibrosis exhibits activity intolerance related to the pulmonary problems
associated with his disease. However, he needs to be encouraged to participate in daily
physical exercise. The ultimate aim of exercise is to:

A. Create a sense of well-being and self-worth
B. Help him overcome respiratory infections
C. Establish an effective, habitual breathing pattern
D. Promote normal growth and development
Answer: C
Explanation:
(A) Regular exercise does promote a sense of well-being and self-worth, but this is not the
ultimate goal of exercise for this client.
(B) Regular chest physiotherapy, not exercise per se, helps to prevent respiratory infections.
(C) Physical exercise is an important adjunct to chest physiotherapy. It stimulates mucus
secretion, promotes a feeling of well-being, and helps to establish a habitual breathing
pattern.
(D) Along with adequate nutrition and minimization of pulmonary complications, exercise
does help promote normal growth and development. However, exercise is promoted primarily
to help establish a habitual breathing pattern.
QUESTION 336
The nurse is caring for a client with pancreatitis. Which of the following IV medications
would the nurse expect the physician to prescribe for control of pain in this client?
A. Morphine sulfate
B. Kerolac tromethamine (Toradol)
C. Promethazine (Phenergan)
D. Meperidine (Demerol)
Answer: D
Explanation:
(A) Morphine sulfate is contraindicated in clients with pancreatitis because it may cause
spasms of the sphincter of Oddi and increase pancreatic pain.
(B) Ketorolac tromethamine is currently not approved by the Food and Drug Administration
for IV use.
(C) Promethazine is a medication that has no analgesic properties.
(D) Meperidine is the drug of choice for clients with pancreatitis. It will not cause spasms at
the sphincter of Oddi, which can lead to increased pancreatic pain.

QUESTION 337
At her first prenatal visit, a 21-year-old woman who is gravida 2, para 0, ab 1, is currently at
32 week’s gestation and has a history of drug abuse, smoking, and occasional ethyl alcohol
use. Fetal ultrasound tests indicate poor fetal growth. The most likely reason for the infant’s
intrauterine growth retardation is:
A. The client’s young age
B. The client’s previous abortion
C. The client’s history of drug, ethyl alcohol, and tobacco use
D. The client’s late prenatal care
Answer: C
Explanation:
(A) Although adolescents frequently have a higher incidence of low-birth-weight infants, this
client is 21 years old.
(B) Uncomplicated induced abortions have not been proved to influence the growth of infants
of subsequent pregnancies.
(C) Compounds in cigarettes and some illicit drugs cause maternal vasoconstriction and a
subsequent reduction in O2 availability for the fetus owing to the resulting reduction in
uteroplacental blood flow. As few as one or two drinks of alcohol per day will decrease birth
weight.
(D) Although early prenatal care has been shown to improve pregnancy outcomes, not
seeking care until the second week of gestation does not, in and of itself, cause intrauterine
growth retardation.
QUESTION 338
A 25-year-old lawyer who is married with three young children works long hours in an effort
to become a partner in the law firm. Following a recent hospitalization for a bleeding ulcer, he
was referred for therapy to treat this psychophysiological disorder. On meeting with the
therapist, he informed him or her that he was a busy man and did not have much time for this
“psych stuff”. When guiding the client to ventilate his feelings, the therapist can expect him
to express feelings of:
A. Guilt
B. Shame
C. Despair
D. Anger

Answer: D
Explanation:
(A) Guilt relates to depression.
(B) Shame is not associated with psychophysiological disorders.
(C) Despair relates to depression.
(D) Repressed anger is associated with psychophysiological disorders.
QUESTION 339
A client on the infectious disease unit is discussing transmission of human immunodeficiency
virus (HIV). The nurse would need to provide more client education based on which client
statement?
A. “HIV is a virus transmitted by sexual contact.”
B. “Condoms reduce the transmission of HIV.”
C. “HIV is a virus that is easily transmitted by casual contact.”
D. “HIV can be transmitted to an unborn infant.”
Answer: C
Explanation:
(A) HIV is transmitted through unprotected sexual contact.
(B) Condoms are an effective barrier to prevent HIV transmission.
(C) HIV is not easily transmitted by casual contact.
(D) HIV can be transmitted intrauterinely at the time of delivery, and by breast-feeding.
QUESTION 340
A 70-year-old female client is admitted to the medical intensive care unit with a diagnosis of
cerebrovascular accident (CVA). She is semi-comatose, responding to pain and change in
position. She is unable to speak or cough. In planning her nursing care for the first 24 hours
following a CVA, which nursing diagnosis should receive the highest priority?
A. Ineffective airway clearance related to immobility, ineffective cough, and decreased level
of consciousness
B. Altered cerebral tissue perfusion related to pathophysiological changes that decrease blood
flow
C. Potential for injury related to impaired mobility and seizures
D. Impaired verbal communication related to aphasia
Answer: A

Explanation:
(A) An effective airway is necessary to prevent hypoxia and subsequent cardiac arrest.
(B) Cerebral tissue perfusion is necessary to preserve remaining cerebral tissue, but this goal
is secondary to maintenance of an effective airway.
(C) While prevention of injury is important, it is secondary to maintaining an effective airway
and cerebral tissue perfusion.
(D) Impaired verbal communication is not life threatening in the acute phase of recovery. It is
the lowest priority of the nursing diagnoses listed.
QUESTION 341
A client had a transurethral resection of the prostate yesterday. He is concerned about the
small amount of blood that is still in his urine. The nurse explains that the blood in his urine:
A. Should not be there on the second day
B. Will stop when the Foley catheter is removed
C. Is normal and he need not be concerned about it
D. Can be removed by irrigating the bladder
Answer: C
Explanation:
(A) Some hematuria is usual for several days after surgery.
(B) The client will continue to have a small amount of hematuria even after the Foley catheter
is removed.
(C) Some hematuria is usual for several days after surgery. The client should not be
concerned about it unless it increases.
(D) Irrigating the bladder will not remove the hematuria. Irrigation is done to remove blood
clots and facilitate urinary drainage.
QUESTION 342
A 28-year-old client performs a long, involved ritual in getting up and preparing for the day.
He became unable to get to his job before noon. His family, in desperation, has admitted him
to the hospital’s psychiatric unit. On the unit, he is always late for breakfast, which is served
at 8 am. The nurse identifies that the best approach to this problem is to:
A. Allow him to eat late
B. Suggest that he do the rituals after breakfast
C. Get him up early so that he can complete the ritual before breakfast

D. Ask him to get all the other clients up so that he will forget about his ritual
Answer: C
Explanation:
(A) Allowing him to eat late is not a solution to the problem because the ritual affects more
than just this meal.
(B) He is helpless to change this behavior because the rituals occur as a result of an irrational
effort to control his anxiety.
(C) To interfere with the ritual will increase anxiety. Until the basic problem is resolved, and
in turn his need for the ritual relieved, arrange the schedule so that essential activities may be
included (such as meals with the group).
(D) This approach would be very disruptive to the other clients and would not serve to relieve
the anxiety of the client.
QUESTION 343
A client is diagnosed with Mycobacterium tuberculosis. He is placed in respiratory isolation,
intubated, and receives mechanical ventilation. When performing suctioning, the nurse
should:
A. Suction for a maximum of 20 seconds
B. Hyper oxygenate before and after suctioning
C. Suction for a maximum of 30 seconds
D. Maintain clean technique during suctioning
Answer: B
Explanation:
(A) The maximum time for suctioning is 10–15 seconds.
(B) Supplemental O2 should be administered before and after suctioning to reduce hypoxia.
(C) The maximum time for suctioning is 10–15 seconds.
(D) Strict sterile technique should be used during suctioning.
QUESTION 344
A 67-year-old postoperative TURP client has hematuria. The nurse caring for him reviews his
postoperative orders and recognizes that which one of the following prescribed medications
would best relieve this problem?
A. Acetaminophen suppository 650 mg
B. Meperidine 50 mg IM

C. Promethazine 25 mg IM
D. Aminocaproic acid (Amicar) 6 g/24 hr
Answer: D
Explanation:
(A) Acetaminophen (Tylenol) has analgesic and antipyretic actions approximately equivalent
to those of aspirin. It produces analgesia possibly by action on the peripheral nervous system.
It reduces fever by direct action on the hypothalamus heat-regulating center with consequent
peripheral vasodilation. It is generally used for temporary relief of mild to moderate pain,
such as a simple headache, minor joint and muscle pains, and control of fever.
(B) Meperidine is a narcotic agonist analgesic with properties similar to morphine except that
it has a shorter duration of action and produces less depression of urinary retention and
smooth muscle spasm. It is used for moderate to severe pain, for a preoperative medication,
for support of anesthesia, and for obstetrical analgesia. In a postoperative TURP client, it
would be used in conjunction with other medications for relief of moderate to severe pain, but
not specifically for bladder spasms associated with TURP surgery.
(C) Promethazine hydrochloride is an antihistamine, antiemetic preparation. It exerts ant
serotonin, anticholinergic, and local anesthetic actions. It is used for symptomatic relief of
various allergic conditions, motion sickness, nausea, and vomiting. It is used for preoperative,
postoperative, and obstetrical sedation and as an adjunct to analgesics for control of pain.
(D) This answer is correct because aminocaproic acid is prescribed specifically for hematuria.
Aminocaproic acid is excreted in the urine. The nurse should be alert for possible signs of
thrombosis, particularly in the extremities.
QUESTION 345
Before completing a nursing diagnosis, the nurse must first:
A. Write goals and objectives
B. Perform an assessment
C. Plan interventions
D. Perform evaluation
Answer: B
Explanation:
(A) Goals and objectives are based on a nursing assessment and diagnosis.
(B) Assessment is the first step of nursing process.
(C) Interventions are nursing actions to meet goals and objectives.

(D) Evaluation process follows nursing interventions.
QUESTION 346
A 22-year-old client presents with a diagnosis of antisocial personality disorder and a history
of using drugs, writing numerous checks with insufficient funds, and stealing. He appears
charming and intelligent, and the other clients are impressed and want to be liked by him. The
greatest problem that may arise from this situation is that:
A. He will manipulate the other clients for his own benefit
B. He will cause the other clients to become psychotic
C. He will become delusional and hallucinate as a result of the excess attention given to him
by peers
D. He may exhibit self-mutilative behavior
Answer: A
Explanation:
(A) This answer is correct. Persons with antisocial personality disorder typically are very
manipulative.
(B) This answer is incorrect. The client’s behavior cannot cause another person to become
psychotic.
(C) This answer is incorrect. Psychosis is not a symptom of antisocial personality. One of the
criteria for diagnosis of this disorder is that no psychosis be present. In addition, the client
would love the attention.
(D) This answer is incorrect. Self-mutilative behavior is characteristic of the borderline
personality disorder.
QUESTION 347
A 45-year-old client has a permanent colostomy. Which of the following foods should he
avoid?
A. Peanut butter and jelly sandwich and milk
B. Corn beef and cabbage and boiled potatoes
C. Oatmeal, whole-wheat toast, and milk
D. Tuna on whole-wheat bread and iced tea
Answer: B
Explanation:
(A, C, D) These foods are allowed with a colostomy.

(B) Gas forming foods such as cabbage should be avoided.
QUESTION 348
A client presents to the emergency room with cyanosis, coughing, tachypnea, and
tachycardia. She has a history of asthma. Arterial blood gas values are pH 7.28, PaO2 54,
PaCO2 60, and HCO3 24. The nursing assessment of arterial blood gases indicate the presence
of:
A. Respiratory alkalosis
B. Respiratory acidosis
C. Metabolic alkalosis
D. Metabolic acidosis
Answer: B
Explanation:
(A) Respiratory alkalosis is determined by elevated pH and low PaCO2.
(B) Respiratory acidosis is determined by low pH and elevated PaCO2.
(C) Metabolic alkalosis is determined by elevated pH and HCO3.
(D) Metabolic acidosis is determined by low pH and HCO3.
QUESTION 349
A 55-year-old client is admitted with a diagnosis of renal calculi. He presented with severe
right flank pain, nausea, and vomiting. The most important nursing action for him at this time
is:
A. Intake and output measurement
B. Daily weights
C. Straining of all urine
D. Administration of O2 therapy
Answer: C
Explanation:
(A) Intake and output measurements are important but must be accompanied by straining
urine.
(B) Daily weights would not provide for identification of calculi.
(C) Straining urine provides for assessment of calculi and evaluation of calculi descent
through ureters and urethra.
(D) O2 therapy should not be necessary for renal calculi.

QUESTION 350
A client returned to the unit following a pneumonectomy. As the nurse is assessing her
incision, she notices fresh blood on the dressing. The nurse should first:
A. Reinforce the dressing.
B. Continue to monitor the dressing.
C. Notify the physician.
D. Note the time and amount of blood.
Answer: C
Explanation:
(A) The dressing should not be reinforced without first notifying the physician. The decision
may be made by the physician to reinforce the dressing after assessing the amount of
bleeding.
(B) Blood on the dressing is unusual and should make the nurse aware that something more
than continuing to monitor the dressing should be done.
(C) The physician should be notified immediately, because if the bleeding persists, the client
may have to be taken back to surgery.
(D) The time and amount of blood do need to be recorded after the physician is notified.
QUESTION 351
A 5-year-old child cries continually in her bed. Her parents have been unsuccessful in
assisting her in expressing her feelings. Which activity should the nurse provide the child to
assist her in expressing her feelings?
A. Books with colorful pictures
B. Music
C. Riding toys
D. Puppets
Answer: D
Explanation:
(A) Books increase cognition, assist with fine motor skills, and augment language
development.
(B) Music provides auditory stimulation and large-muscle activity.
(C) Riding toys provide large-muscle activity.

(D) Puppets allow expression of feelings and fears that otherwise could not be directly
communicated.
QUESTION 352
Which of the following blood gas parameters primarily reflects respiratory function?
A. PCO2
B. CO2 content of the blood
C. HCO3
D. Base excess
Answer: A
Explanation:
(A) The lungs are responsible for regulation of CO2, and this parameter primarily reflects
respiratory function.
(B) CO2 content of the blood is an indirect measure of respiratory function.
(C) HCO3 is a measure of kidney function only and is important in acid-base balance.
(D) Base excess represents the excess of HCO3 and is not reflective of respiratory function.
QUESTION 353
A 2-day-old infant boy has been diagnosed with an atrial septal defect due to a persistent
patent foramen ovale. When explaining the diagnosis to the mother, the nurse includes in the
discussion the function of the foramen ovale. In fetal circulation, the foramen ovale allows a
portion of the blood to bypass the:
A. Left ventricle
B. Pulmonary system
C. Liver
D. Superior vena cava
Answer: B
Explanation:
(A) The foramen ovale permits a percentage of the blood to shunt from the right atrium to the
left atrium. The blood then goes to the left ventricle, permitting systemic fetal circulation with
blood containing a higher O2 saturation.
(B) As the blood shunts from the right atrium to the left atrium, the pulmonary system is
bypassed. The fetus receives O2 from the maternal circulation, thereby permitting the partial
bypass of the pulmonary system.

(C) The foramen ovale is located in the atrial septum of the heart and does not affect the liver.
(D) The superior vena cava returns blood to the heart, bringing blood to the location of the
foramen ovale.
QUESTION 354
A 27-year-old primigravida stated that she got up from the chair to fix dinner and bright red
blood was running down her legs. She denies any pain previously or currently. The client is
very concerned about whether her baby will be all right. Her vital signs include P 120 bpm,
respirations 26 breaths/min, BP 104/58 mm Hg, temperature 98.2°F, and fetal heart rate 146
bpm. Laboratory findings revealed hemoglobin 9.0 g/dL, hematocrit 26%, and coagulation
studies within normal range. On admission, the peri pad she wore was noted to be half
saturated with bright red blood. A medical diagnosis of placenta previa is made. The priority
nursing diagnosis for this client would be:
A. Decreased cardiac output related to excessive bleeding
B. Potential for fluid volume excess related to fluid resuscitation
C. Anxiety related to threat to self
D. Alteration in parenting related to potential fetal injury
Answer: A
Explanation:
(A) Based on the client’s history, presence of bright red vaginal bleeding, and hemoglobin
value on admission, the priority nursing diagnosis would be decreased cardiac output related
to excessive bleeding.
(B) This nursing diagnosis is a potential problem that does not exist at the present time, and
therefore is not the priority problem.
(C) The client’s expressed anxiety is for her child. The fetus will remain physiologically safe
if the decreased cardiac output is resolved.
(D) Initial spontaneous bleeding with placenta previa is rarely life threatening to the mother
or the fetus. Delivery of the fetus will be postponed until fetal maturity is achieved and
survival is likely.
QUESTION 355
Home-care instructions for the child following a cardiac catheterization should include:
A. Notify the physician if a slight bruise develops around the insertion site.
B. Use sponge bathing until stitches are removed.

C. Give aspirin if the child complains of pain at the insertion site.
D. Keep a clean, dry dressing on the insertion site for 2 days.
Answer: B
Explanation:
(A) A small bruise may develop around the insertion site and is not a reason for alarm.
(B) It is best to keep the child out of the bathtub until the sutures are removed.
(C) Acetaminophen, not aspirin, is the drug of choice if there is pain at the insertion site.
(D) The insertion site should be kept clean and dry and open to air.
QUESTION 356
The nurse is caring for a client who has had a tracheostomy for 7 years. The client is started
on a full strength tube feeding at 75 mL/hr. Prior to starting the tube feeding, the nurse
confirms placement of the tube in the stomach. The hospital policy states that all tube feeding
must be dyed blue. On suctioning, the nurse notices the sputum to be a blue color. This is
indicative of which of the following?
A. The client aspirated tube feeding.
B. The nurse has placed the suction catheter in the esophagus.
C. This is a normal finding.
D. The feeding is infusing into the trachea.
Answer: A
Explanation:
(A) Once the feeding tube placement is confirmed in the stomach, aspiration can occur if the
client’s stomach becomes too full. When suctioning the trachea, if secretions resemble tube
feeding, the client has aspirated the feeding.
(B) Because the trachea provides direct access to a client’s airway, it would not be possible to
place the catheter in the esophagus.
(C) Blue-colored sputum is never considered a normal finding and should be reported and
documented.
(D) The nurse confirmed placement of the feeding tube in the stomach prior to initiating the
tube feeding; therefore, it is highly unlikely that the feeding tube would be located in the
trachea.

QUESTION 357
A client has returned to the unit following a left femoral popliteal bypass graft. Six hours
later, his dorsalis pedis pulse cannot be palpated, and his foot is cool and dusky. The nurse
should:
A. Continue to monitor the foot
B. Notify the physician immediately
C. Reposition and reassess the foot
D. Assure the client that his foot is fine
Answer: B
Explanation:
(A) The client is losing blood supply to his left foot. Continuing to monitor the foot will not
help restore the blood supply to the foot.
(B) The physician should be notified immediately because the client is losing blood supply to
his left foot and is in danger of losing the foot and/or leg.
(C) The presenting symptoms are of an emergency nature and require immediate intervention.
(D) This action would be giving the client false assurance.
QUESTION 358
A 2-year-old child will undergo a cardiac catheterization tomorrow to evaluate his ventricular
septal defect. Based on his developmental stage, the nurse:
A. Uses pictures to explain the procedure to the child and his parents that evening
B. Explains the procedure using simple words and sentences just before the preoperative
sedation
C. Asks the parents to explain the procedure to the child after she explains it to them
D. Asks the parents to leave the room while the preoperative medication and instructions are
given
Answer: B
Explanation:
(A) A toddler is not capable of conceptualizing about the inside of his body and is concerned
about body intactness; therefore, diagrams would not be useful. Also, the previous evening is
too far from the procedure for the toddler to remember the instructions.
(B) A simple explanation the morning of the procedure is the best developmental strategy to
use, because it focuses on the toddler’s need for parental support, body intactness, and short
attention span.

(C) A relationship between the nurse and the child needs to develop. Also, misinformation
may be given to the child if the parents explain the procedure to the child.
(D) The parents are the child’s support system and need to be there to strengthen the child.
QUESTION 359
A 34-year-old client who is gravida 1, para 0 has a history of infertility and conceived this
pregnancy while taking fertility drugs. She is at 32 week’s gestation and is carrying triplets.
She is complaining of low back pain and a feeling of pelvic pressure. Her cervical exam
reveals a long, closed cervix. The nurse notes that the client is experiencing mild uterine
contractions every 7–8 minutes after the nurse has placed her on the fetal monitor. Her
condition should indicate that:
A. Her cervix shows she will likely deliver soon
B. The nurse should not be alarmed because mild uterine activity is common at 32 week’s
gestation
C. She may be in preterm labor because this is more common with multiple pregnancies
D. She most likely has a urinary tract infection (UTI) because this is common with pregnancy
Answer: C
Explanation:
(A) Her cervical exam is normal. There are no cervical changes at this time.
(B) Braxton Hicks contractions may be common throughout pregnancy, but they are not
regular.
(C) Rhythmical contractions in conjunction with low back pain and pelvic pressure at 32
weeks in a woman carrying triplets are of great concern. She may be in preterm labor.
(D) UTIs are common in pregnancy due to the enlarging uterus compressing the ureters and
the stasis of urine. The woman would be more likely to complain of urinary frequency and
urgency, fever or chills, and malodorous urine with a UTI.
QUESTION 360
Clients receiving antipsychotic drug therapy will often exhibit extrapyramidal side effects that
are reversible with which of the following agents ordered by the physician?
A. Phenothiazines
B. Anticholinergics
C. Anti-Parkinsonian drugs
D. Tricyclic agents

Answer: B
Explanation:
(A) This answer is incorrect. Phenothiazines are antipsychotic drugs and produce the
symptoms.
(B) This answer is correct. Anticholinergic agents are often used prophylactically for
extrapyramidal symptoms. They balance cholinergic activity in the basal ganglia of the brain.
(C) This answer is incorrect. Anti- Parkinsonian drugs would increase the symptoms.
(D) This answer is incorrect. Tricyclic agents are used for symptoms of depression.
QUESTION 361
An 80-year-old widow is living with her son and daughter- in-law. The home health nurse has
been making weekly visits to draw blood for a prothrombin time test. The client is taking 5
mg of coumadin per day. She appears more debilitated, and bruises are noted on her face.
Elder abuse is suspected. Which of the following are signs of persons who are at risk for
abusing an elderly person?
A. A family member who is having marital problems and is regularly abusing alcohol
B. A person with adequate communication and coping skills who is employed by the family
C. A friend of the family who wants to help but is minimally competent
D. A lifelong friend of the client who is often confused
Answer: A
Explanation:
(A) This answer is correct. Two risk factors are identified in this answer.
(B) This answer is incorrect. Persons at risk tend to lack communication skills and effective
coping patterns.
(C) This answer is incorrect. Persons at risk are usually family members or those reluctant to
provide care.
(D) This answer is incorrect. This individual has a vested interest in providing care.
QUESTION 362
The primary focus of nursing interventions for the child experiencing sickle cell crisis is
aimed toward:
A. Maintaining an adequate level of hydration
B. Providing pain relief
C. Preventing infection

D. O2 therapy
Answer: A
Explanation:
(A) Maintaining the hydration level is the focus for nursing intervention because dehydration
enhances the sickling process. Both oral and parenteral fluids are used.
(B) The pain is a result of the sickling process. Analgesics or narcotics will be used for
symptom relief, but the underlying cause of the pain will be resolved with hydration.
(C) Serious bacterial infections may result owing to splenic dysfunction. This is true at all
times, not just during the acute period of a crisis.
(D) O2 therapy is used for symptomatic relief of the hypoxia resulting from the sickling
process. Hydration is the primary intervention to alleviate the dehydration that enhances the
sickling process.
QUESTION 363
A client is to have a coronary artery bypass graft performed in the morning using a saphenous
vein. He wants to know why the physician does not use the internal mammary artery for his
bypass graft because his friend’s physician uses this artery. The nurse tells the client that the
internal mammary artery:
A. Takes more time to remove
B. Has a greater risk of becoming reoccluded
C. Is smaller in diameter
D. Has too many valves
Answer: A
Explanation:
(A) It does take more time to remove the internal mammary artery, and this is one reason why
some physicians do not use it.
(B) There is not a greater risk of reocclusion. In fact, it may actually stay patent longer.
(C) The internal mammary artery is actually larger in diameter than the saphenous vein.
(D) The internal mammary artery does not have too many valves.
QUESTION 364
A client has ascites, which is caused by:
A. Decreased plasma proteins
B. Electrolyte imbalance

C. Decreased renal function
D. Portal hypertension
Answer: A
Explanation:
(A) A decrease in plasma proteins causes a decrease in intravascular osmotic pressure
resulting in leakage of fluid into peritoneal cavity.
(B) Fluid and electrolyte imbalance may occur as a result of the ascites.
(C) Ascites is a result of hepatic malfunction, not renal malfunction.
(D) Portal hypertension causes esophageal varices, not ascites.
QUESTION 365
The most appropriate method of evaluating whether the diet of a child with cystic fibrosis is
meeting his caloric needs is:
A. Careful monitoring of weight loss or gain
B. Carefully recording amounts and types of foods ingested
C. Keeping a strict account of the number of calories ingested
D. Keeping a careful account of the amount of pancreatic enzymes ingested
Answer: A
Explanation:
(A) Consistent weight gain, even if it is slow, is an indication that the child is eating and
digesting sufficient calories.
(B) Recording how much the child eats is useful, but it is not an indicator of how well his
body is using the foods consumed.
(C) Counting calories will indicate how much he is eating, but it will not reflect whether or
not the foods are properly digested.
(D) Keeping track of the enzyme intake will indicate compliance with medication but not
whether the child is getting sufficient calories.
QUESTION 366
A client had a right below-the-knee amputation 4 days ago. He is complaining of pain in his
right lower leg. The nurse should:
A. Remind the client that he no longer has that part of his leg and assure him he will be OK
B. Call the physician to request a psychological consultation for the client
C. Turn on the television to distract the client’s attention from his amputated leg

D. Give the client his order of Demerol 50 mg IM prn
Answer: D
Explanation:
(A) The nurse is ignoring the client’s pain. Telling the client that he will be OK will not
relieve his phantom pain.
(B) The client does not need a psychological consultation. Phantom pain is a normal sensation
experienced by clients with amputations.
(C) Using the television as a distractor will not relieve the client’s phantom pain.
(D) Phantom pain is a normal, very real experience for an amputee and should be treated with
pain medication.
QUESTION 367
A 27-year-old healthy primigravida is brought to the labor and birthing room by her husband
at 32 week’s gestation. She experienced a sudden onset of painless vaginal bleeding.
Following an ultrasound examination, the diagnosis of bleeding secondary to complete
placenta previa is made. Expected assessment findings concerning the abdomen would
include:
A. A rigid, board like abdomen
B. Uterine atony
C. A soft relaxed abdomen
D. Hypertonicity of the uterus
Answer: C
Explanation:
(A) A rigid, board like abdomen is an assessment finding indicative of placenta abruptio.
(B) A cause of post birth hemorrhage is uterine atony. With placenta previa, uterine tone is
within normal range.
(C) The placenta is located directly over the cervical os in complete previa. Blood will escape
through the os, resulting in the uterus and abdomen remaining soft and relaxed.
(D) In placenta abruptio, hypertonicity of the uterus is caused by the entrapment of blood
between the placenta and uterine wall, a retroplacental bleed. This does not exist in placenta
previa.

QUESTION 368
Succinylcholine chloride (Anectine) is ordered prior to electroconvulsive therapy treatment
for depressed clients. The nurse explains that the purpose of the drug is to:
A. Relax muscles
B. Relieve anxiety
C. Reduce secretions
D. Act as an anesthetic
Answer: A
Explanation:
(A) Succinylcholine chloride relaxes muscles and decreases the intensity of the seizure.
(B) Succinylcholine chloride does not relieve anxiety.
(C) Atropine is given to reduce secretions.
(D) Thiamylal sodium (Surital) or other phenobarbital preparations are used as brief
anesthetics.
QUESTION 369
Plans for the care of a client with an ulcer caused by emotional problems need to take into
consideration that:
A. His priority needs are limited to medical management
B. There is no real psychological basis for his illness
C. The disorder is a threat to his physical well-being
D. He is unable to participate in planning his care
Answer: C
Explanation:
(A) There may be a medical emergency that takes top priority; however, the basis of the
problem is emotional.
(B) The problem is a physical manifestation of an emotional conflict.
(C) The bleeding ulcer can be life threatening.
(D) For lifestyle change to occur, the client must participate in the planning of his care so that
he is committed to changes that will have positive results.

QUESTION 370
A client has just been transferred to the floor from the labor and delivery unit following
delivery of a stillborn term infant. She is very despondent. When the nurse attempts to take
her vital signs, she responds in anger, stating, “You leave me alone”. You don’t care anything
about me. It’s people like you who let my baby die. The nurse’s best course of action is to:
A. Quietly leave her room, allowing her more private time to deal with her loss.
B. Tell her that what happened was for the best and that she is still young and can have other
children.
C. Tell her how sorry you are, and let her know that her child is now a little angel in heaven.
D. Tell her how sorry you are about the loss of her baby, and acknowledge her anger as being
a normal stage of grief. Assure her that you are there to help her in any way you can.
Answer: D
Explanation:
(A) Parents do need their privacy following a loss, but the nurse still has a responsibility to
provide postpartum physical care.
(B) This is a negative statement, which is not therapeutic. The client is not concerned about
future children but is in the first stages of grief, denial, and anger.
(C) This is a negative statement, which is not therapeutic. The client does not want to hear
about her baby in heaven. She cannot believe that God could love or want her child more than
she could.
(D) Acknowledging that anger is normal and beneficial will help the client to understand the
normal stages of grief. Expressing sorrow over her loss and assuring her that the support is
there to take care of her physical and emotional needs will help to promote a trusting
relationship.
QUESTION 371
Except for initial explosiveness on admission, a client diagnosed with schizophrenia stays in
her room. She continues to believe other people are out to get her. A nursing intervention
basic to improving withdrawn behavior is:
A. Assigning her to occupational therapy
B. Having her sit with the nurses while they chart
C. Helping her to make friends
D. Facilitating communication
Answer: D

Explanation:
(A) The nurse does not make this assignment.
(B) One-to-one observation is not appropriate. It does not focus on the client or encourage
communication.
(C) The client is too suspicious to accomplish this goal.
(D) The withdrawn individual must learn to communicate on a one-to-one level before
moving on to more threatening situations.
QUESTION 372
A 72-year-old client with a new colostomy is being evaluated at the clinic today for
constipation. When discussing diet with the client, the nurse recognizes that which one of the
following foods most likely caused this problem?
A. Fried chicken
B. Eggs
C. Tapioca
D. Cabbage
Answer: C
Explanation:
(A) Fried, greasy food, such as fried chicken, will produce diarrheal Ike stools in individuals
with all types of GI ostomies.
(B) Eggs will cause odor-producing stools in individuals with all types of GI ostomies.
(C) Tapioca and rice products will cause constipation in individuals with all types of GI
ostomies.
(D) Cabbage will cause odor-producing and flatus-producing stools in individuals with all
types of GI ostomies.
QUESTION 373
Which of the following nursing actions is essential to prevent drug-resistant tuberculosis?
A. Monitor liver function.
B. Monitor renal function.
C. Assess knowledge of respiratory isolation.
D. Monitor compliance with drug therapy.
Answer: D
Explanation:

(A) Monitoring liver function will not prevent the development of drug-resistant organisms.
(B) Monitoring renal function will not prevent the development of drug-resistant organisms.
(C) Knowledge of respiratory isolation will reduce transmission of tuberculosis but will not
prevent development of drug-resistant organisms.
(D) Noncompliance with prescribed antituberculosis drug regimen is the primary cause of
drug-resistant organisms. Noncompliance permits the mutation of organisms.
QUESTION 374
A successful executive left her job and became a housewife after her marriage to a plastic
surgeon. She started doing volunteer work for a charity organization. She developed pain in
her legs that advanced to the point of paralysis. Her physicians can find no organic basis for
the paralysis. The client’s behavior can be described as:
A. Housework phobia
B. Malingering
C. Conversion reaction
D. Agoraphobia
Answer: C
Explanation:
(A) A typical phobia does not result in physical symptoms (i.e., paralysis).
(B) Malingering is pretending to be ill. This person has a true paralysis.
(C) A conversion reaction is a physical expression of an emotional conflict. It has no organic
basis.
(D) Agoraphobia is fear of public places.
QUESTION 375
The nurse needs to be aware that the most common early complication of a myocardial
infarction is:
A. Diabetes mellitus
B. Anaphylactic shock
C. Cardiac hypertrophy
D. Cardiac dysrhythmia
Answer: D
Explanation:
(A) Diabetes mellitus is not a common complication of myocardial infarction.

(B) Anaphylactic shock is an allergic reaction.
(C) Cardiac hypertrophy is a late potential complication. It is a common complication of
congestive heart failure.
(D) Myocardial infarction causes tissue damage, which may interrupt electrical impulses.
Myocardial irritability results from lack of oxygenated tissue.
QUESTION 376
Several months after antibiotic therapy, a child is readmitted to the hospital with an
exacerbation of osteomyelitis, which is now in the chronic stage. The mother appears anxious
and asks what she could have done to prevent the exacerbation. The nurse’s response is based
on the knowledge that chronic osteomyelitis:
A. Is caused by poor physical conditions or poor nutrition
B. Often results from unhygienic conditions or an unclean environment
C. Is directly related to sluggish circulation in the affected limb
D. May develop from sinuses in the involved bone that retain infectious material
Answer: D
Explanation:
(A) Poor nutrition and/or poor physical conditions are factors that predispose to the
development of osteomyelitis but do not cause it.
(B) An unclean or unhygienic environment may predispose to the development of chronic
osteomyelitis, but it does not cause an exacerbation of the previous infection.
(C) Sluggish circulation through the medullary cavity during acute osteomyelitis may delay
healing, but it does not cause the disease to become chronic.
(D) Areas of sequestrum may be surrounded by dense bone, become honeycombed with
sinuses, and retain infectious organisms for a long time.
QUESTION 377
The nurse is notified that a 27-year-old primigravida diagnosed with complete placenta previa
is to be admitted to the hospital for a cesarean section. The client is now at 36 week’s
gestation and is presently having bright red bleeding of moderate amount. On admission, the
nursing intervention that the nurse should give the highest priority to is:
A. Shave the client’s abdomen and arrange her lab work
B. Determine the status of the fetus by fetal heart tones
C. Start an IV infusion in the client’s arm

D. Insert an indwelling catheter into her bladder
Answer: B
Explanation:
(A) These nursing actions are necessary prior to the cesarean section, but not immediately
necessary to maintain physiological equilibrium.
(B) Determining the physiological status of the fetus would constitute the highest priority in
evaluating and maintaining fetal life.
(C) These nursing actions are necessary prior to the cesarean section, but not immediately
necessary to maintain physiological equilibrium.
(D) These nursing actions are necessary prior to the cesarean section, but not immediately
necessary to maintain physiological equilibrium.
QUESTION 378
The parents of a 2-year-old child are ready to begin toilet training activities with him. His
parents feel he is ready to train because he is now 2 years old. What would the nurse identify
as readiness in this child?
A. Patience by the child when wearing soiled diapers
B. Communicating the urge to defecate or urinate
C. The child awakening wet from his naps
D. The age at which the child’s siblings were trained
Answer: B
Explanation:
(A) Children experience impatience with soiled diapers when readiness for training is
apparent. They often desire to be changed immediately.
(B) A child must be able to use verbal or nonverbal skills to communicate needs.
(C) A readiness indicator would be awaking dry from naps.
(D) The age at which a sibling was toilet trained has no implications for training this child.
QUESTION 379
The 4th of July holiday comes while a client is in the hospital being treated for schizophrenia.
She is taking chlorpromazine and has improved to the point of being allowed to go with a
group to the park for a picnic. The side effect of chlorpromazine that the nurse needs to keep
in mind during this outing is:
A. Hypotension

B. Photosensitivity
C. Excessive appetite
D. Dryness of the mouth
Answer: B
Explanation:
(A) A decrease in blood pressure sometimes occurs with chlorpromazine. It would not be a
factor influenced by a picnic in the park.
(B) Protection from the sun is important in clients taking phenothiazines because they burn
easily and severely.
(C) An appetite increase sometimes occurs with chlorpromazine. It would not be affected by a
picnic.
(D) Dryness of the mouth may occur at any time and is not affected by the picnic outing.
QUESTION 380
Parents of children receiving chemotherapy should be warned that alopecia is a side effect
and that:
A. Children seldom show concern about losing their hair
B. The hair will come out gradually, and the loss will not be noticeable for some time
C. It is best for girls to choose a wig similar to their hair style and color before the hair falls
out
D. The parents will soon get used to seeing their children without hair, and it will no longer
bother them
Answer: C
Explanation:
(A) Children may become depressed with a changed appearance and not want to look at
themselves or have others see them.
(B) The hair will fall out in clumps, causing patchy baldness that is quite noticeable and
traumatic to children and their families.
(C) Having a wig that looks like a girl’s own hair can be a psychological boost to children and
is helpful in fostering later adjustments to hair loss.
(D) Families may become accustomed to seeing their children without hair, but the loss is
traumatic to them and will continue to bother them.

QUESTION 381
A client had abdominal surgery this morning. The nurse notices that there is a small amount
of bloody drainage on his surgical dressing. The nurse would document this as what type of
drainage?
A. Serosanguinous
B. Purulent
C. Sanguinous
D. Catarrhal
Answer: C
Explanation:
(A) Drainage from a surgical incision usually proceeds from sanguinous to serosanguinous.
(B) Purulent drainage usually indicates infection and should not be seen initially from a
surgical incision.
(C) Drainage from a surgical incision is initially sanguinous, proceeding to serosanguinous,
and then to serous.
(D) Catarrhal is a type of exudate seen in upper respiratory infections, not in surgical
incisions.
QUESTION 382
A neonate was admitted to the hospital with projectile vomiting. According to the parents, the
baby had experienced vomiting episodes after feeding for the last 2 days. A medical diagnosis
of hypertrophic pyloric stenosis was made. On assessment, the infant had poor skin turgor,
sunken eyeballs, dry skin, and weight loss. Identify the number-one priority nursing
diagnosis.
A. Fluid volume deficit
B. Altered nutrition
C. Altered bowel elimination
D. Anxiety
Answer: A
Explanation:
(A) Fluid volume deficit is the major problem. Symptoms of dehydration are evident. The
effects of fluid and electrolyte balance may be life threatening. Rehydration can be
accomplished effectively through IV fluids and electrolytes.

(B) Vomiting may also signal a nutritional problem. However, the nutritional problem would
be secondary to fluid and electrolyte disturbances. The infant may also be placed on NPO
status.
(C) With vomiting, a decrease in the size and number of stools is expected.
(D) The infant cannot verbalize feelings of anxiety. Anxiety would not be an appropriate
diagnosis.
QUESTION 383
A client has developed congestive heart failure secondary to his myocardial infarction.
Discharge diet instructions should emphasize the reduction or avoidance of:
A. Fresh vegetables and fruit
B. Canned vegetables and fruit
C. Breads, cereals, and rice
D. Fish
Answer: B
Explanation:
(A) Fresh vegetables and fruits are excellent sources of essential vitamins.
(B) Canned and frozen foods have a high sodium content. Labels of all canned foods should
be read to determine if sodium is used in any form.
(C) Bread, cereal, and rice are excellent sources of carbohydrates.
(D) Fish is an excellent source of protein.
QUESTION 384
A depressed client is seen at the mental health center for follow-up after an attempted suicide
1 week ago. She has taken phenelzine sulfate (Nardil), a monoamine oxidase (MAO)
inhibitor, for 7 straight days. She states that she is not feeling any better. The nurse explains
that the drug must accumulate to an effective level before symptoms are totally relieved.
Symptom relief is expected to occur within:
A. 10 days
B. 2–4 weeks
C. 2 months
D. 3 months
Answer: B
Explanation:

(A) This answer is incorrect. It can take up to 1 month for therapeutic effect of the
medication.
(B) This answer is correct. Because MAO inhibitors are slow to act, it takes 2–4 weeks before
improvement of symptoms is noted.
(C) This answer is incorrect. It can take up to 1 month for therapeutic effect of the
medication.
(D) This answer is incorrect. Therapeutic effects of the medication are noted within 1 month
of drug therapy.
QUESTION 385
A client’s behavior is annoying other clients on the unit. He is meddling with their belongings
and dominating the group. The best approach by the nurse is to:
A. Seclude him in his room.
B. Set limits on his behavior.
C. Have his medication increased.
D. Ignore him and tell the other clients that these behaviors are due to his illness and that they
should understand.
Answer: B
Explanation:
(A) This action by the nurse would be punitive.
(B) Consistent limit setting will help the client to know what is acceptable behavior.
(C) This action is not within the nurse’s scope of practice.
(D) This could be dangerous to the client and to others and violates other client’s rights.
QUESTION 386
A client is being treated for congestive heart failure. His medical regimen consists of digoxin
(Lanoxin) 0.25 mg po daily and furosemide 20 mg po bid. Which laboratory test should the
nurse monitor?
A. Intake and output
B. Calcium
C. Potassium
D. Magnesium
Answer: C
Explanation:

(A) Intake and output are not laboratory tests.
(B) Serum calcium levels are not affected by digoxin or furosemide.
(C) Furosemide is a non–potassium-sparing loop diuretic. Hypokalemia is a common side
effect of furosemide and may enhance digoxin toxicity.
(D) Serum magnesium levels are not affected by digoxin or furosemide.
QUESTION 387
A male client is diagnosed with hypoparathyroidism. He has been on dialysis for several
years. He is experiencing symptoms such as numbness of the lips, muscle weakness,
carpopedal spasms, and wheezing. Given the client’s symptoms, nursing assessment would
focus on:
A. Detection of tetany
B. Detection of hypocalcemia to prevent seizures
C. Evidence of depression
D. Detection of premature cataract formation
Answer: A
Explanation:
(A) Assessment should focus on detection of tetany, which is the most common symptom of
hypoparathyroidism. Left undetected and untreated, tetany resulting from hypocalcemia can
progress to seizures.
(B) Hypocalcemia is difficult to detect on nursing assessment alone. Abdominal cramping
may be an indication of hypocalcemia, but laboratory data are required to confirm diagnosis.
(C) Depression can be a symptom of hypoparathyroidism, but it is not definitive.
(D) Premature cataract formation can occur, but it also is not specific to parathyroidism and
poses no immediate danger to the client.
QUESTION 388
A client is placed on lithium therapy for her manic-depressive illness. When monitoring the
client, the nurse assesses the laboratory blood values. Toxicity may occur with lithium
therapy when the blood level is above:
A. 1.0 mEq/L
B. 2.2 mEq/L
C. 0.03 mEq/L
D. 1.5 mEq/L

Answer: D
Explanation:
(A) This value is a low blood level.
(B) This value is a toxic blood level.
(C) This value is a low blood level.
(D) This value is the level at which most clients are maintained, and toxicity may occur if the
level increases. The client should be monitored closely for symptoms, because some clients
become toxic even at this level.
QUESTION 389
A 17-year-old pregnant client who is gravida 1, para 0, is at 36 week’s gestation. Based on the
nurse’s knowledge of the maternal physiological changes in pregnancy, which of these
findings would be of concern?
A. Complaints of dyspnea
B. Edema of face and hands
C. Pulse of 65 bpm at 8 weeks, 73 bpm at 36 weeks
D. Hematocrit 39%
Answer: B
Explanation:
(A) Dyspnea is a common complaint during the third trimester owing to the increasing size of
the uterus and the resulting pressure against the diaphragm.
(B) Edema of the face, hands, or pitting edema after 12 hours of bed rest may be indicative of
preeclampsia and would be of great concern to the healthcare provider.
(C) An increase in heart rate of 10–15 bpm is a normal physiological change in pregnancy
due to the multiple hemodynamic changes.
(D) A hematocrit value of 39% is within the normal range. A value 35% would indicate
anemia.
QUESTION 390
A 14-year-old client has a history of lying, stealing, and destruction of property. Personal
items of peers have been found missing. After group therapy, a peer approaches the nurse to
report that he has seen the 14- year-old with some of the missing items. The best response of
the nurse is to:
A. Request that he explain to the group why he took personal items from peers

B. Approach him when he is alone to inquire about his involvement in the incident
C. Imply to him that you doubt his involvement in the incident and request his denial
D. Confront him openly in group and request an apology
Answer: B
Explanation:
(A) This answer is incorrect. There is no proof that he removed the missing items.
(B) This answer is correct. Anxiety and defensiveness are lessened if the individual is
approached in this manner.
(C) This answer is incorrect. It is difficult for one to admit to wrongdoing with this approach.
(D) This answer is incorrect. He has not yet been proved guilty. Confrontation will only
increase defensiveness and anxiety.
QUESTION 391
A 5-year-old child is hospitalized for an acute illness. The nurse encourages the family to
bring her favorite objects from home. What is the nurse’s rationale?
A. To reduce fear of the unknown
B. To keep the child calm
C. To establish a trusting relationship
D. To prevent or minimize separation anxiety
Answer: D
Explanation:
(A) Objects from home do not reduce fear of the unknown. Children need explanations,
reassurance, and preparation for the unknown. Also, parental presence can promote comfort
and feelings of security.
(B) A calm, relaxed, and reassuring manner will assist in calming the child. The child’s
objects from home will not assist in calming the child.
(C) A trusting relationship is based on the quality of the nurse-client relationship. Objects
from home have no impact.
(D) Favorite objects from home assist in creating a familiar setting. Also, these objects may
prevent or minimize separation from the child’s usual routine and family support.
QUESTION 392
A male client has experienced low back pain for several years. He is the primary support of
his wife and six children. Although he would qualify for disability, he plans to continue his

employment as long as possible. His back pain has increased recently, and he is unable to
control it with non-steroidal anti-inflammatory agents. He refuses surgery and cannot take
narcotics and remain alert enough to concentrate at work. His physician has suggested
application of a transcutaneous electrical nerve stimulation (TENS) unit. Which of the
following is an appropriate rationale for using a TENS unit for relief of pain?
A. TENS units have an ultrasonic effect that relaxes muscles, decreases joint stiffness, and
increases range of motion.
B. TENS units produce endogenous opioids that affect the central nervous system with
analgesic potency comparable to morphine.
C. TENS units work on the gate-control theory of pain; bio stimulation therapy of large fibers
block painful stimuli.
D. TENS units prevent muscle spasms, decrease the potential for further injury, and minimize
pressure on joints.
Answer: C
Explanation:
(A) TENS units do not have this effect, but whirlpool therapy does.
(B) TENS units do not produce endogenous opioids, only the body can do that.
(C) TENS units do work based on the gate control theory of pain control.
(D) TENS units do not have this effect, but possibly changing the client’s position would.
QUESTION 393
A client was admitted to the hospital for a TURP. Within 48 hours of admission and 12 hours
postoperatively, both the blood pressure and pulse increased. He became agitated, thought
snakes were crawling on his arms and legs, and generally became unmanageable. He pulled
out his IV and urinary catheter in attempt to rid himself of the snakes. He was sweating
profusely. The admission nurse’s notes indicated that the client admitted to “having a few
drinks now and then.” He is probably experiencing which of the following?
A. Major psychotic depression
B. Delirium tremens
C. Generalized anxiety disorder
D. Adjustment disorder with mixed features
Answer: B
Explanation:

(A) Symptoms of psychotic depression must exist for at least 2 weeks, and the symptoms
must represent a change from previous functioning.
(B) Delirium tremens occur approximately on the second or third day following cessation or
reduction of alcohol intake. Symptoms would be all those described in the situation.
(C) Symptoms exhibited by this client are not exhibited in clients with anxiety disorders, who
manifest excessive or unrealistic worry about life circumstances for at least 6 months.
(D) Symptoms for adjustment disorders with mixed emotional features (e.g., depression and
anxiety) are different from those exhibited by the client in this situation.
QUESTION 394
The mother of a 7-year-old mental health center client reports that the client has refused to
attend gymnastics for the past 2 weeks. Prior to that time, the child liked going to this class
and was attending 3 times a week. In talking with the client, the nurse would:
A. Ask her why she doesn’t like gymnastics anymore
B. Ask her to describe how things were at gymnastics before she started refusing to go
C. Tell her that it is OK to be afraid of this activity
D. Reassure her that things will get better once she begins the classes again
Answer: B
Explanation:
(A) The child has not said that she dislikes gymnastics.
(B) The nurse will be able to obtain information on what events occurred at gymnastics prior
to her refusal to attend. The nurse will also gain information about the child’s perception of
the problem.
(C) The child has not said she is afraid to go to gymnastics.
(D) False reassurance is inappropriate.
QUESTION 395
A 68-year-old client developed acute respiratory distress syndrome while hospitalized for
pneumonia. After a respiratory arrest, an endotracheal tube was inserted. Several days later,
numerous attempts to wean him from mechanical ventilation were ineffective, and a
tracheostomy was created. For the first 24 hours following tracheostomy, it is important to
minimize bleeding around the insertion site. The nurse can accomplish this by:
A. Deflating the cuff for 10 minutes every other hour instead of 5 minutes every hour
B. Avoiding manipulation of the tracheostomy including cuff deflation

C. Reporting any signs of crepitus immediately to the physician
D. Changing tracheostomy dressing only as necessary using one-half strength hydrogen
peroxide to cleanse the site
Answer: B
Explanation:
(A) The tracheal cuff should not be deflated within the first 24 hours following surgery.
(B) To minimize bleeding, any manipulation, including cuff deflation, should be avoided.
(C) Small amounts of crepitus are expected to occur; however, large amounts or expansion of
the area of crepitus should be reported to the physician.
(D) The tracheostomy site may be changed as often as necessary, but site care should be done
with normal saline.
QUESTION 396
A 60-year-old woman exhibits forgetfulness, emotional lability, confusion, and decreased
concentration. She has been unable to perform activities of daily living without assistance.
After a thorough medical evaluation, a diagnosis of Alzheimer’s disease was made. An
appropriate nursing intervention to decrease the anxiety of this client would include:
A. Allowing the client to perform activities of daily living as much as possible unassisted
B. Confronting confabulations
C. Reality testing
D. Providing a highly stimulating environment
Answer: A
Explanation:
(A) This answer is correct. The more the client is able to control her daily routine, the less
anxiety she will experience.
(B) This answer is incorrect. Confrontation tends to increase anxiety.
(C) This answer is incorrect. Reality testing is an assessment tool. It does not decrease
anxiety.
(D) This answer is incorrect. A highly stimulating environment increases distractibility and
anxiety.
QUESTION 397
A 22-year-old single woman was admitted to the psychiatric hospital by her mother, who
reported bizarre behavior. Except for going to work, she spends all her time in her room and

expresses concern over neighbors spying on her. She has fears of the telephone being
“bugged”. Her diagnosis is schizophrenia. One nurse per shift is assigned to work with the
client. The primary reason for this plan would be to:
A. Protect her from suicide
B. Enable her to develop trust
C. Supervise her medication regimen
D. Involve her in groups for social interaction
Answer: B
Explanation:
(A) Suicide is a greater risk in depression than in schizophrenia.
(B) The client is suspicious and needs help to develop trust, which is basic to her
improvement.
(C) Although she will be taking medication, drug therapy would not necessitate consistency
in the nurses assigned.
(D) A suspicious client should have limited exposure to groups, because group participation
increases discomfort.
QUESTION 398
The serial sevens test is often used to determine delirium and dementia. This test aids in
assessing which of the following?
A. Abstract thinking
B. Ability to focus and concentrate thoughts
C. Judgment
D. Memory
Answer: B
Explanation:
(A) This answer is incorrect. The test measures the abilities to concentrate and calculate. The
use of proverbs is the most common way to test abstraction.
(B) This answer is correct. The serial sevens test is a common test of calculation ability. It is
difficult for the demented or delirious client to perform.
(C) This answer is incorrect. The test for judgment should predict whether the individual will
behave in a socially accepted manner.
(D) This answer is incorrect. In testing memory, the nurse would attempt to get the client
either to recall recent events or to think about past events.

QUESTION 399
A mother called the physician’s office to ask if it would help relieve her small daughter’s
abdominal pain if she gave an enema and placed a heating pad on the abdomen. Her daughter
has a fever and has vomited twice. The nurse’s response is based on the knowledge that:
A. The symptoms could easily have been caused by constipation, which an enema would
relieve
B. Heat would help to relax the abdominal muscles and relieve her pain
C. Both heat and enemas stimulate intestinal motility and could increase the risk of
perforation
D. Complaints of stomach ache are common in young children and are generally best ignored
Answer: C
Explanation:
(A) Constipation does not cause fever or vomiting but may cause anorexia. Risk of
perforation outweighs the possible benefits of an enema.
(B) Heat will not relieve her symptoms but will increase intestinal motility and increase the
risk of perforation.
(C) Heat and enemas are contraindicated where severe abdominal pain is suspected because
they increase intestinal motility and the risk of perforation.
(D) Complaints accompanied by physical symptoms such as pain, anorexia, and fever should
never be ignored.
QUESTION 400
A 58-year-old client on a general surgery unit is scheduled for transurethral resection of the
prostate (TURP) in 2 hours. The nurse explains to the client that this procedure means:
A. Removal of the prostate tissue by way of a lower abdominal midline incision through the
bladder and into the prostate gland
B. Removal of prostate tissue by a resectoscope that is inserted through the penile urethra
C. Removal of the prostate tissue by an open surgical approach through an incision between
the ischial tuberosities, the scrotum, and the rectum
D. Removal of prostate tissue by an open surgical approach through a low horizontal incision,
bypassing the bladder, to the prostate gland
Answer: B
Explanation:

(A) This describes a suprapubic (trans vesical) prostatectomy procedure.
(B) This is the correct description of a TURP procedure.
(C) This describes a perineal prostatectomy procedure.
(D) This describes a retropubic (extravesical) prostatectomy procedure.

Document Details

Related Documents

person
Harper Mitchell View profile
Close

Send listing report

highlight_off

You already reported this listing

The report is private and won't be shared with the owner

rotate_right
Close
rotate_right
Close

Send Message

image
Close

My favorites

image
Close

Application Form

image
Notifications visibility rotate_right Clear all Close close
image
image
arrow_left
arrow_right